Sunteți pe pagina 1din 70

SEMANGAT MY LOVELY, LOVE YOU

FOREVER AND EVER TILL THE END *>*


KUMPULAN SOAL – SOAL
ATLS
BAHASA INGGRIS &
INDONESIA

Microsoft Office User


[COMPANY NAME]
SOAL POST TEST a. administer 0-negative blood
b. applyextemal warming devices.
1. Cardiac tamponade after trauma c. Control internal hemorrhage operatively
a. is seldom life-threatening d. apply the pneumatic antishock garment
b. can be excluded by an upright, AP chest x-ray e. infuse large volumes of intravenous crystalloid solution.
c. can be confused with a tension pneumothorax
d. causes a fall in systolic pressure of > 15 mm Hg with expiration 5. To establish a diagnosis of shock,
e. most commonly occurs after blunt injury to the anterior chest wall a. systolic blood pressure must be below 90 mm Hg.
b. the presence of a closed head injury should be excluded
2. Which one of the following statements regarding patients with thoracic spine c. acidosis should be present by arterial blood \gas analysis
injuries is TRUE? d. the patient must fail to respond to intravenous fluid infusion.
a. Log-rolling may be destabilizing to fractures from T-12 to L-1. e. clinical evidence of inadequate organ perfusion must be present.
b. Adequate immobilization can be accomplished with the scoop stretcher.
c. Spinal cord injury below T-10 usually spares bowel and bladder function. 6. A 23-year-old man is brought immediately to the emergency department from
d. Hyperflexion fractures in the upper thoracic spine are inherently the hospital' s parking lot where he was shot in the lower abdomen.
unstable. Examination reveals a single bullet wound. He is breathing and has a thready
e. These patients rarely present with spinal shock in association with cord pulse. However, he is unconscious and has no detectable blood pressure.
injury. Optimal immediate management is to
a. perform diagnostic peritoneal lavage.
3. Absence of breath sounds and dullness to percussion over the left b. initiate infusion of packed red blood cells.
hemithorax are fmdings best explained by c. insert a nasogastric tube and urinary catheter.
a. Left hemothorax. d. transfer the patient to the operating room, while initiating fluid
b. cardiac contusion therapy.
c. left simple pneumothorax e. initiate fluid therapy to return his blood pressure to normotensive
d. left diaphragmatic rupture
e. right tension pneumothorax. 7. An electrician is electrocuted by a downed power line after a thunderstorm.
He apparently made contact with the wire at the level of the right mid thigh.
4. A young man sustains a gunshot wound to the abdomen and is brought In the emergency department, his vital signs are normal and no dysrhythmia
promptly to the emergency department by prehospital personnel. His skin is is noted on ECG. On examination, there is an exit wound on the bottom of
cool and diaphoretic, and he is confused. His pulse is thready and his femoral the right foot. His urine is positive for blood by dip stick but no RBCs are seen
pulse is only weakly palpable. The defmitive treatment in managing this microscopically. Initial management should include
patient is to a. immediate angiography.
2
b. aggressive fluid infusion. a. inserting an oropharyngealairvvay.
c. intravenouspyleography. b. inserting a nasopharyngeal airway.
d. debridement of necrotic muscle. c. performing a surgical cricothyroidotomy.
e. admission to the intensive care unit for observation. d. performingfiberoptic-guided nasotracheal intubation.
e. performingorotracheal intubation after obtaining a lateral c-spine x-ray.
8. An 8-year-old girl is an unrestrained passenger in a vehicle struck from
behind. In the emergency department, her blood pressure is 80/60 mm Hg, 11. The primary indication for transferring a patient to a higher level trauma
heart rate is 80 beats per minute, and respiratory rate is 16 breaths per center is
minute. Her GCS score is 14. She complains that her legs feel "funny and a. unavailability of a surgeon or operating room staff.
won't move right;" however, her spine x-rays do not show a fracture or b. multiple system injuries, including severe head injury.
dislocation. A spinal cord injury in this child c. resource limitations as determined by the transferring doctor.
a. is most likely a central cord syndrome. d. resource limitations as determined by the hospital administration.
b. must be diagnosed by magnetic resonance imaging. e. widened mediastinum on chest x-ray following blunt thoracic trauma.
c. can be excluded by obtaining a CT of the entire spine.
d. may exist in the absence of objective findings on x-ray studies. 12. A young man sustains a ritle wound to the mid-abdomen. He is brought
e. is unlikely because of the incomplete calcification of the vertebral bodies. promptly to the emergency department by prehospital personnel. His skin is
cool and diaphoretic, and his systolic blood pressure is 58 rnm Hg. Warmed
9. Immediate chest tube insertion is indicated for which of the following crystalloid fluids are initiated without improvement in his vital signs. The next,
conditions? most appropriate step is to perform
a. Pneumothorax a. celiotomy.
b. Pneumomediastinum b. an abdominal CT scan.
c. Massive hemothorax c. diagnostic laparoscopy.
d. Diaphragmatic rupture d. abdominal ultrasonography.
e. Subcutaneous emphysema e. a diagnostic peritoneal lavage.

10. A 32-year-old man is brought to the hospital unconscious with severe facial 13. A teen-aged bicycle rider is hit by a truck traveling at a high rate of speed. In
injuries and noisy respirations after an automobile collision. In the emergency the emergency department, she is actively bleeding from open fractures of
department, he has no apparent injury to the anterior aspect of his neck. He her legs, and has abrasions on her chest and abdominal wall. Her blood
suddenly becomes apneic, and attempted ventilation with a face mask is pressure is 80/50 mm Hg, heart rate is 140 beats per minute, respiratory rate
unsuccessful. Examination of his mouth reveals a large hematoma of the is 8 breaths per minute, and GCS score is 6. The first step in managing this
pharynx with loss of normal anatomic landmarks. Initial management of his patient is to
airway should consist of a. obtain a lateral cervical spine x-ray.
3
b. insert a central venous pressure line. 16. Which of the following statements regarding injury to the central nervous
c. administer 2 liters of crystalloid solution. system in children is TRUE?
d. perform endotracheal intubation and ventilation. a. Children suffer spinal cord injury without x-ray abnormality more
e. apply the PASG and inflate the leg compartments. commonly than adults.
b. An infant with a traumatic brain injury may become hypotensive from
14. An 8-year-old boy falls 4.5 meters (15 feet) from a tree and is brought to the cerebral edema.
emergency department by his family. His vital signs are normal, but he c. Initial therapy for the child with traumatic brain injury includes the
complains of left upper quadrant pain. An abdominal CT scan reveals a administration of methylprednisolone intravenously.
moderately severe laceration of the spleen. The receiving institution does not d. Children have more focal mass lesions as a result of traumatic brain
have 24-hour-a-day operating room capabilities. The most appropriate injury when compared to adults .
management of this patient would be to e. Young children are less tolerant of expanding intracranial mass lesions
a. type and crossmatch for blood. than adults.
b. request consultation of a pediatrician.
c. transfer the patient to a trauma center. 17. During an altercation, a 32-year-old man sustains a gunshot wound to the
d. admit the patient to the intensive care unit. right upper hemithorax, above the nipple line with an exit wound posteriorly
e. prepare the patient for surgery the next day. above the scapula on the right. He is transported by ambulance to a
community hospital. He is endotracheally intubated, closed tube
15. A 17-year-old helmeted motorcyclist is struck broadside by an automobile at thoracostomy is performed, and 2 liters of Ringer's lactate solution are
an intersection. He is unconscious at the scene with a blood pressure of infused through 2 large-caliber IVs. His blood pressure now is 60/0 mm Hg,
140/90 mm Hg, heart rate of 90 beats per minute, and respiratory rate of 22 heart rate is 160 beats per minute, and respiratory rate is 14 breaths per
breaths per minute. His respirations are sonorous and deep. His GCS score minute (ventilated with 100% 02). The most appropriate next step in
is 6. Immobilization of the entire patient may include the use of all the managing this patient is
following EXCEPT a. celiotomy.
a. air splints. b. diagnostic peritoneal lavage.
b. bolstering devices. c. arterial blood gas determination.
c. a long spine board. d. administer packed red blood cells.
d. a scoop-style stretcher. e. chest x-ray to confinn tube placement.
e. A semirigid cervical collar.
18. A 42-year-old man, injured in a motor vehicle crash, suffers a closed head
injury, multiple palpable left rib fractures, and bilateral femur fractures. He is
intubated orotracheally without difficulty. Initially, his ventilations are easily
assisted with a bag¬valve device. It becomes more difficult to ventilate the
4
patient over the next 5 minutes, and his hemoglobin oxygen saturation level d. This is not a problem in the traumatized Rh-positive pregnant patient.
decreases from 98% to89 % . The most appropriate next step is to e. initiation of Rh immunoglobulin therapy does not require proof of
a. obtain a chest x-ray. fetomaternal hemorrhage.
b. decrease the tidal volume.
c. auscultate the patient's chest. 22. A 30-year-old man is struck by a car traveling at 56 kph (35 mph). He has
d. increase the rate of assisted ventilations. obvious fractures of the left tibia near the knee, pain in the pelvic area, and
e. perform needle decompression of the left chest. severe dyspnea. His heart rate is 180 beats per minute, and his respiratory
rate is 48 breaths per minute with no breath sounds heard in the left chest. A
19. A 24-year-old woman passenger in an automobile strikes the wind screen tension pneumothorax is relieved by immediate needle decompression and
with her face during a head-on collision. In the emergency department, she tube thoracostomy. Subsequently, his heart rate decreases to 140 beats per
is talking and has marked facial edema and crepitus. The highest priority minute, his respiratory rate decreases to 36 breaths per minute, and his blood
should be given to pressure is 80/50 inm Hg. Warmed Ringer's lactate is administered
a. lateral, c-spine x-ray. intravenously. The next priority should be to:
b. upper airway protection. a. perform a urethrogram and cystogram.
c. carotid pulse assessment. b. perform external fixation of the pelvis.
d. management of blood loss. c. obtain abdominal and pelvic CT scans.
e. determination of associated Injuries. d. perform arterial embolization of the pelvic vessels.
e. perform diagnostic peritoneal lavage or abdominal ultrasound.
20. Twenty-seven patients are seriously injured in an aircraft accident at a local
airport. The basic principle of triage should be to 23. Regarding shock in the child, which of the following is FALSE?
a. treat the most severely injured patients first. a. Vital signs are age-related.
b. establish a field triage area directed by a doctor. b. Children have greater physiologic reserves than do adults.
c. rapidly transport all patients to the nearest appropriate hospital. c. Tachycardia is the primary physiologic response to hypovolemia.
d. treat the greatest number of patients in the shortest period of time. d. The absolute volume of blood loss required to produce shock is the
e. produce the greatest number of survivors based on available same as in adults.
resources. e. An initial fluid bolus for resuscitation should approximate 20 mL/kg of
Ringer's lactate
21. Which one of the following statements is FALSE concerning Rh
isoimmunization in the pregnant trauma patient? 24. A 30-year-old man sustains a severely comminuted, open distal right femur
a. It occurs in blunt or penetrating abdominal trauma. fracture in a motorcycle crash. The wound is actively bleeding. Normal
b. Minor degrees of fetomaternal hemorrhage produce it. sensation is present over the lateral aspect of the foot but decreased over the
c. A negative Kleihauer-Betke test excludes Rh isoimmunization. medial foot and great toe. Normal motion of the foot is observed.
5
Dorsalispedis and posterior tibial pulses are easily palpable on the left, but b. Anticoagulants
heard only by Doppler on the right. Immediate efforts to improve circulation c. Warm (40°C) water
to the injured extremity should involve d. Padding and elevation
a. immediate angiography. e. Topical application of silvasulphadiazine
b. tamponade of the wound with a pressure dressing.
c. wound exploration and removal of bony fragments. 28. The driver of a single car crash is orotracheally intubated in the field by
d. realignment of the fracture segments with a traction splint. prehospital personnel after they identify a closed head injury and determine
e. fasciotomy of all four compartments in the lower extremity. that the patient is unable to protect his airway. In the emergency department,
the patient demonstrates decorticate posturing bilaterally. He is being
25. A crosstable, lateral x-ray of the cervical spine ventilated with a bag-valve device, but his breath sounds are absent in the
a. must precede endotracheal intubation. left hemithorax. His blood pressure is 160/88 mm Hg, heart rate is 70 beats
b. excludes serious cervical spine injury. per minute, and the pulse oximeter displays a hemoglobin oxygen saturation
c. is an essential part of the primary survey. of 96% . The next step in assessing and managing this patient should be to
d. is not necessary for unconscious patients with penetrating cervical a. determine the arterial blood gases.
injuries. b. obtain a lateral cervical spine x-ray.
e. is unacceptable unless 7 cervical vertebrae and the C-7 to T-1 c. assess placement of the endotracheal tube.
relationship are visualized. d. perform needle decompression of the left chest.
e. insert a thoracostomy tube in the left hemithorax.
26. An 18-year-old, helmeted motorcyclist is brought by ambulance to the
emergency department following a high-speed crash. Prehospitalpersormel 29. Early central venous pressure monitoring during fluid resuscitation in the
report that he was thrown 15 meters (50 feet) off his bfice. He has a history emergency department has the greatest utility in a
of hypotension prior to arrival in the emergency department, but is now a. patient with a splenic laceration.
awake, alert, and conversational. Which of the following statements is TRUE? b. patient with an inhalation injury.
a. Cerebral perfiision is intact c. 6-year-old child with a pelvic fracture.
b. Intravascular volume status is normal. d. patient with a severe cardiac contusion.
c. The patient has sensitive vasomotor reflexes. e. 24-year-old man with a massive hemothorax.
d. Intraabdominal visceral injuries are unlikely.
e. The patient probably has an acute epidural hematoma. 30. The response to catecholamines in an injured, hypovolemic pregnant woman
can be expected to result in
27. Which one of the following is the recommended method for initially treating a. placental abruption.
frostbite? b. fetal hypoxia and distress.
a. Vasodilators c. fetal/maternal dysrhythmia.
6
d. improved uterine blood flow. his lower extremities is detected, even in response to painful stimuli. The
e. increased maternal renal blood flow. most likely cause for this fmding is
a. an epidural hematoma.
31. A 5-year-old boy is struck by an automobile and brought to the emergency b. a pelvic fracture.
department. He is lethargic, but withdraws purposefully from painful stimuli. c. central cord syndrome.
His blood pressure is 90 mm Hg systolic, heart rate is 140 beats per minute, d. intracerebral hemorrhage.
and his respiratory rate is 36 breaths per minute. The preferred route of e. bilateral compartment syndrome.
venous access in this patient is
a. percutaneous femoral vein cannulation 34. All of the following signs on the chest x-ray of a blunt injury victim may
b. cutdown on the saphenous vein at the ankle. suggest aortic rupture EXCEPT:
c. intraosseous catheter placement in the proximal tibia. a. mediastinal emphysema.
d. percutaneous peripheral veins in the upper extremities. b. presence of a "pleural cap."
e. central venous access via the subclavian or interna1 jugular vein. c. obliteration of the aortic knob.
d. deviation of the trachea to the right.
32. A 22-year-old man is brought to the hospital after crashing his motorcycle into e. depression of the left mainstem bronchus
a telephone pole. He is unconscious and in profound shock. He has no open
wounds or obvious fractures. The cause of his shock is MOST LIKELY 35. A young woman sustains a severe head injury as the result of a motor
caused by vehicular crash. In the emergency department, her GCS score is 6. Her blood
a. a subdural hematoma. pressure is 140/90 mm Hg and her heart rate is 80 beats per minute. She is
b. an epidural hematoma. intubated and is being mechanically ventilated. Her pupils are 3 mm in size
c. a transected lumbar spinal cord. and equally reactive to light. There is no other apparent injury. The most
d. a transected cervical spinal cord. important principle to follow in the early management of her head injury is to
e. hemorrhage into the chest or abdomen. a. administer an osmotic diuretic.
b. prevent secondary brain injury.
33. A 42-year-old man is trapped from the waist down beneath his overtumed c. aggressively treat systemic hypertension.
tractor for several hours before medical assistance arrives. He is awake and d. reduce metabolic requirements of the brain.
alert until just before arriving in the emergency department. He is now e. distinguish between intracranial hematoma and cerebral edema.
unconscious and responds only to painful stimuli by moaning. His pupils are
3 mm in diameter and symmetrically reactive to light. Prehospital personnel 36. A 25-year-old woman is brought to the emergency department after a motor
indicate that they have not seen the patient move either of his lower vehicle crash. She was initially lucid at the scene and then developed a
extremities. On examination in the emergency department, no movement of dilated pupil and contralateral extremity weakness. In the emergency

7
department, she is unconscious and has a GCS score of 6. The initial c. Blood pressure
management step for this patient should be to d. Urinary output
a. obtain a CT scan of the head. e. Jugular venous pressure
b. administerdecadron 20 mg IV.
c. perform endotracheal intubation. 40. Which one of the following physical findings suggests a cause of hypotension
d. initiate an W line and administer Mannitol 1 g/kg. other than spinal cord injury?
e. perform an emergency linar hole on the side of the dilated pupil. a. priapism.
b. bradycardia.
37. Contraindication to nasogastric intubation is the presence of a c. diaphragmatic breathing.
a. gastric perforation. d. presence of deep tendon reflexes.
b. diaphragmatic rupture. e. ability to flex forearms but inability to extend them.
c. open depressed skull fracture.
d. fracture of the cervical spine.
e. fracture of the cribriform plate. SOAL INITIAL

38. A 24-year-old man sustains multiple fractured ribs bilaterally as a result of In managing the head-injured patient, the most important initial step is to:
being crushed in a press at a plywood factory. Examination in the emergency a. secure the airway
department reveals a flail segment of the patient's thorax. Primary h. obtain a c-spine film
resuscitation includes high-flow oxygen administration via a nonrebreathing c. support the circulation
mask, and initiation of Ringer' s lactate solution. The patient exhibits d. control scalp hemorrhage
progressive confusion, cyanosis, and tachypnea. Management at this time e. determine the GCS score
should consist of
a. intravenous sedation. A previously healthy, 70-kg man suffers an estimated acute blood loss of 2 liters.
b. external stabilization of the chest wall. Which one of the following statements applies to this patient?
c. increasing the F102 in the inspired gas. a. His pulse pressure will be widened.
d. intercostal nerve blocks for pain relief. b. His urinary output will be at the lower limits of normal.
e. endotracheal intubation and mechanical ventilation. c. He will have tachycardia, but no change in his systolic blood pressure.
d. His systolic blood pressure will be decreased with a narrowed pulse pressure.
39. During resuscitation, which one of the following is the most reliable as a guide e. His systolic blood pressure will be maintained with an elevated diastolic pressure.
to volume replacement?
a. Pulse rate
b. Hematocrit
8
The diagnosis of shock must include: d.9
a. hypoxemia e. 12
b. acidosis
c. Tachycardia Which one of the following findings in an adult is most likely
d. increased vascular resistance to require immediate management during the primary
e. evidence of inadequate organ perfusion survey?
a. distended abdomen
A 7-year-old boy is brought to the emergency department by his parents several b. Glasgow Coma Scale score of 11
minutes after he c. temperature of 36.5°C (97.8°F)
fell through a window. He is bleeding profusely from a 6-cm wound of his medial right d. deforming of the right thigh
thigh. Immediate management of the wound should consist of: e. respiratory rate of 40 breaths per minute
a. application of a tourniquet
b. direct pressure on the wound The following are contraindications for tetanus toxoid administration
c. packing the wound with gauze a. history of neurological reaction or severe hypersensitivity to the product
d. direct pressure on the femoral artery at the groin b. Local side effects
e. debridement of devitalized tissue c. muscular spasms
d. pregnancy
The first maneuver to improve oxygenation after chest injury is: e. all of the above
a. intubate the patient
b. assess arterial blood gases A 56-year-old man is thrown violently against the steering wheel of his truck during a
c. administer supplemental oxygen motor vehich crash. On arrival in the emergency department he is diaphoretic and
d. ascertain the need for a chest tube complaining of chest pain. His blood pressure is 60/40 mm Hg and his respiratory rate
e. obtain a chest x-ray is 40 breaths per minute. Which of the following best differentiates cardiac tamponade
from tension pneumothorax as the cause of his hypotension?
A 25-year-old man, injured in a motor vehicular crash, is admitted to the emergency a. tachycardia
department. His pupils react sluggishly and his eyes open to painful stimuli. He does b. pulse volume
not follow commands, but he does moan periodically. His right arm is deformed and c. breath sounds
does not respond to painful stimulus; however, his left hand reaches purposefully d. pulse pressure
toward the painful stimulus. Both legs are stiffly extended. His GCS score is: e. jugular venous pressure
a. 2
b. 4
c. 6
9
A 39-year-old man is admitted to the emergency department after an automobile A young woman sustains a severe head injury as the result of a motor vehicle crash.
collision. He is cyanotic, has insufficient respiratory effort, and has a GCS score of 6. In the emergency department, her GCS is 6. Her blood pressure is 140/90 mm Hg
His full beard makes it difficult to fit the oxygen facemask to his face. The most and her heart rate is 80 beats per minute. She is intubated and mechanically
appropriate next step is to ventilated. Her pupils are 3 mm in size and equally reactive to light. There is no other
a. perform a surgical cricothyroidotomy apparent injury. The most important principle to follow in the early management of her
b. attempt nasotracheal intubation head injury is to:
c. ventilate him with a bag-mask device until c-spine injury can be excluded a. avoid hypotension
d. attempt orotracheal intubation using 2 people and inline stabilization of the b. administer an osmotic diuretic
cervical spine c. aggressively treat systemic hypertension
e. ventilate the patient with a bag-mask device until his beard can be shaved for d. reduce metabolic requirements of the brain
better mask fit e. distinguish between intracranial hematoma and cerebral edema

A 32-year-old man's right leg is trapped beneath his overturned car for nearly 2 hours A 33-year-old woman is involved in a head-on motor vehicle crash. It took 30 minutes
before he is extricated. On arrival in the emergency department,his right lower to extricate her from the car. Upon arrival in the emergency department, her heart rate
extremity is cool, mottled, insensate, and motionless. Despite normal vital signs, is 120 beats per minute, BP is 90/70 mm Hg, respiratory rate is 16 breaths per minute,
pulses cannot be palpated below thefemoral vessel and the muscles of the lower and her GCS score is 15. Examination reveals bilaterally equal breath sounds,
extremity are firm and hard. During the management of this patient, which of the anterior chest wall ecchymosis, and distended neck veins. Her abdomen is flat, soft,
following is most likely to improve the chances for limb salvage? and not tender. Her pelvis is stable. Palpable distal pulses are found in all 4
a. applying skeletal traction extremities. Of the following, the most likely diagnosis is:
b. administering anticoagulant drugs a. hemorrhagic shock
c. administering thrombolytic therapy b. cardiac tamponade
d. perform right lower extremity fasciotomy c. massive hemothorax
e. transferring the patient to the trauma center 120 km away d. tension pneumothorax
e. diaphragmatic rupture
Hemorrhage of 20% of the patient's blood volume is associated usually with:
a. oliguria A trauma patient presents to your emergency department with inspiratory stridor and
b. confusion a suspected c-spine injury. Oxygen saturation is 88% on high-flow oxygen via a
c. hypotension nonrebreathing mask. The most appropriate next step is to:
d. tachycardia a. apply cervical traction
e. blood transfusion requirement b. perform immediate tracheostomy
c. insert bilateral thoracostomy tubes
d. maintain 100% oxygen and obtain immediate c-spine x-rays
10
e. maintain inline immobilization and establish a definitive airway

The principle of balanced resuscitation is


A 22-year-old man is hypoeinnvit and tam after s shows) ward to *a lea slemslier
 His
blood mow is Wank 80/40 mm Hs, After 2 Inas of crystaliosd salaam his blood pinixc a. permissive hypotension and early plasma infusion
increases to 122.'10 nun Hi_ His heart rate is now 100 boats per minim sad has b. equal amounts of crystalloid and colloids
 c.
reapsransry o , breath,
 * per numne. His breads sounds are deceased in the left c. simultaneous management of breathing and circulation
hawthorn. mod after magi IV amid resuscitation, a closed tube thomoonomy is d. maintenance of a normal acid base balance e.
performed for deceased left breath soon& *ledge return of as small amount of blood e. achieving a pulse rate <90
and no air leak. After chest tube insertion, the mast appraprela next step is
In managing the head-injured patient, the most important initial step is to
a. reexamine the chest 

a. secure the airway

b. perform an sonogram 

b. obtain a c-spine film
c. obtain a CT scan of the chest 
 obtain arterial blood gas analyses
 c. support the circulation
d. control scalp hemorrhage
d. perform transesophageal echocardiography 

e. determine the GCS score

A construction worker falls two stories from a building and sustams bilateral cake:nal A previously healthy, 70-kg (154-pound) man suffers an estimated acute blood loss
fm In the emergency department, he is alert, vital signs are noting, and he is of 2 liters. Which one of the following statements applies to this patient?
complaining of severe pain in both heels and his lower back. Lower extremity pulses
are strong and that is no other deformity. The suspected diagnosis is most likely to be a. His pulse pressure will be widened.

confirmed by
b. His urinary output will be at the lower limits of normal.

a. angiography
c. He will have tachycardia, but no change in his systolic blood pressure.

b. compartment pressures 

d. His systolic blood pressure will be decreased with a narrowed pulse
c. retrograde urethrogram 
 pressure.

d. Doppler ultrasound studies 
 e. His systolic blood pressure will be maintained with an elevated diastolic
pressure.
e. complete spine x-ray series 

The physiologic hypervolemia of pregnancy has clinical significance in the

11
management of the severely injured, gravid woman by: d. direct pressure on the femoral artery at the groin
e. debridement of devitalized tissue
a. reducing the need for blood transfusion 

For the patient with severe traumatic brain injury, profound hypocarbia should be
b. increasing the risk of pulmonary edema 
 avoided to prevent:
c. complicating the management of closed head injury
d. reducing the volume of crystalloid required for resuscitation a. respiratory acidosis 

e. increasing the volume of blood loss to produce maternal hypotension
b. metabolic acidosis 

The best assessment of fluid resuscitation of the burn patient is: c. cerebral vasoconstriction with diminished perfusion 

a. adequate urinary output d. neurogenic pulmonary edema 

b. reversal of systemic acidosis
e. shift of the oxyhemoglobin dissociation curve 

c. normalization of the heart rate
d. normal central venous pressure
 A 25-year-old man is brought to a hospital with a general surgeon after being involved
e. 4 mL/kg/percent body burn/24 hours in a motor vehicle crash. Computed tomography shows an aortic injury and splenic
laceration with free abdominal fluid. His blood pressure falls to 70 mm Hg after CT.
The diagnosis of shock must include: The next step is:
a. hypoxemia
a. contrast angiography

b. acidosis
c. hypotension b. transfer to a higher level trauma center
d. increased vascular resistance
 c. exploratory laparotomy

e. evidence of inadequate organ perfusion d. infuse additional crystalloids
e. transesophageal echocardiography
A 7-year-old boy is brought to the emergency department by his parents several
minutes after he fell through a window. He is bleeding profusely from a 6-cm wound Which one of the following statements regarding abdominal trauma in the pregnant
of his medial right thigh. Immediate management of the wound should consist of: patient is TRUE?

a. application of a tourniquet a. The fetus is in jeopardy with major abdominal trauma.



b. direct pressure on the wound
b. Leakage of amniotic fluid is an indication for hospital admission.

c. packing the wound with gauze
c. Indications for peritoneal lavage are different from those in the nonpregnant
12
patient. b. insert an oropharyngeal airway

d. Penetration of an abdominal hollow viscus is more common in late than in
c. perform needle decompression of the right chest
early pregnancy.
d. manually displace the gravid uterus to the left side of the abdomen
e. The secondary survey follows a different pattern from that of the nonpregnant
e. initiate 2, large-caliber peripheral IV lines and crystalloid infusion
patient.
Which one of the following findings in an adult is most likely to require immediate
The first maneuver to improve oxygenation after chest injury is:
management during the primary survey?

a. intubate the patient



a. distended abdomen

b. assess arterial blood gases
b. Glasgow Coma Scale score of 11
c. administer supplemental oxygen
c. temperature of 36.5°C (97.8°F)
d. ascertain the need for a chest tube
e. obtain a chest x-ray d. deforming of the right thigh

e. respiratory rate of 40 breaths per minute
A 25-year-old man, injured in a motor vehicular crash, is admitted to the emergency
department. His pupils react sluggishly and his eyes open to painful stimuli. He does The most important, immediate step in the management of an open pneumothorax is:
not follow commands, but he does moan periodically. His right arm is deformed and a. endotracheal intubation

does not respond to painful stimulus; however, his left hand reaches purposefully
toward the painful stimulus. Both legs are stiffly extended. His GCS score is: b. operation to close the wound

c. placing a chest tube through the chest wound
a. 2 d. placement of an occlusive dressing over the wound
b. 4 e. initiation of 2, large-caliber IVs with crystalloid solution
c. 6
d. 9 The following are contraindications for tetanus toxoid administration
e. 12
a. history of neurological reaction or severe hypersensitivity to the product
A 20-year-old woman, at 32 weeks gestation, is stabbed in the upper right chest. In b. Local side effects

the emergency department, her blood pressure is 80/60 mm Hg. She is gasping for
breath, extremely anxious, and yelling for help. Breath sounds are diminished in the c. muscular spasms

right chest. The most appropriate first step is to: d. pregnancy
e. all of the above
a. perform tracheal intubation

A 56-year-old man is thrown violently against the steering wheel of his truck during a
13
motor vehich A
 crash. On arrival in the emergency department he is diaphoretic and mL of blood has drained from the right chest. The most appropriate next step in
complaining of chest pain. His blood pressure is 60/40 mm Hg and his respiratory rate managing this patient is to
is 40 breaths per minute. Which of the following best differentiates cardiac tamponade
from tension pneumothorax as the cause of his hypotension? a. perform FAST

b. obtain a CT of the chest

a. tachycardia
b. pulse volume c. perform an angiography

c. breath sounds d. urgently transfer the patient to the operating room
d. pulse pressure e. immediately transfer the patient to a trauma center
e. jugular venous pressure
A 39-year-old man is admitted to the emergency department after an automobile
collision. He is cyanotic, has insufficient respiratory effort, and has a GCS score of 6.
His full beard makes it difficult to fit the oxygen facemask to his face. The most
appropriate next step is to
Bronchial intubation of the right or left mainstem bronchus can easily occur during
infant endotracheal intubation because: a. perform a surgical cricothyroidotomy

b. attempt nasotracheal intubation

a. The trachea is relatively short.

c. ventilate him with a bag-mask device until c-spine injury can be excluded

b. The distance from the lips to the larynx is relatively short.

d. attempt orotracheal intubation using 2 people and inline stabilization of the
c. The use of tubes without cuffs allows the tube to slip distally.
 cervical spine
d. The mainstem bronchi are less angulated in their relation to the trachea. e. ventilate the patient with a bag-mask device until his beard can be shaved for
e. So little friction exists between the endotracheal tube and the wall of the better mask fit
trachea.
A patient is brought to the emergency department after a motor vehicle crash. He is
A 23-year-old man sustains 4 stab wounds to the upper right chest during an conscious and there is no obvious external trauma. He arrives at the hospital
altercation and is brought by ambulance to a hospital that has full surgical capabilities. completely immobilized on a long spine board. His blood pressure is 60/40 mm Hg
His wounds are all abovr the nipple. He is endotracheally intubated, closed tube and his heart rate is 70 beats per minute. His skin is warm. Which one of the following
thoracostomy is performed, and 2 liters of crystalloid solution are infused through 2 statements is TRUE?
large-caliber IVs. His blood pressure now is 60/0 mm Hg, heart rate is 160 beats per
minute, and respiratory rate is 14 breaths per minute (ventilated with 100% 02). 1500 a. Vasoactive medications have no role in this patient's management.


14
b. The hypotension should be managed with volume resuscitation alone. a. request a CT scan

c. Flexion and extension views of the c-spine should be performed early.
d. Abdominal visceral injuries can be excluded as a cause of hypotension. b. insert a gastric tube

e. Flaccidity of the lower extremities and loss of deep tendon reflexes are c. suction the oropharynx

expected.
d. obtain a lateral cervical spine x-ray
Which one of the following is the most effective method for initially treating frostbite? e. ventilate the patient with a bag-mask

a. moist heat
 A
 22-year-old man sustains a gunshot wound to the left chest and is transported to
b. early amputation a small community hospital at which surgical capabilities are not available. In the
c. padding and elevation emergency department, a chest tube is inserted and 700 mL, of blood is evacuated.
d. vasodilators and heparin The trauma center accepts the patient in transfer. Just before the patient is placed in
e. topical application of silver sulfadiazine the ambulance for transfer, his blood pressure decreases to 80/68 mm Hg and his
heart rate increases to 136 beats per minute. The next step should be to:
A 32-year-old man's right leg is trapped beneath his overturned car for nearly 2 hours
before he is extricated. On arrival in the emergency department,his right lower a. clamp the chest tube

extremity is cool, mottled, insensate, and motionless. Despite normal vital signs,
b. cancel the patient's transfer

pulses cannot be palpated below the femoral vessel and the muscles of the lower
extremity are firm and hard. During the management of this patient, which of the c. perform an emergency department thoracotomy
following is most likely to improve the chances for limb salvage? d. repeat the primary survey and proceed with transfer
e. delay the transfer until the referring doctor can contact a thoracic surgeon
a. applying skeletal traction
b. administering thrombolytic therapy
c. administering anticoagulant drugs
64-year-old man is involved in a high-speed car crash, is resuscitated initially in a
d. perform right lower extremity fasciotomy
 small hospital with limited resources. He has a closed head injury with a GCS score
e. transferring the patient to the trauma center 120 km away of 13. He has a widened mediastinum on chest x-ray with fractures of left ribs 2
through 4, but no pneumothorax. After infusing 2 liters of crystalloid solution, his blood
A patient arrives in the emergency department after being beaten about the head and pressure is 110/74 mm Hg, heart rate is 100 beats per minute, and respiratory rate is
face with a wooden club. He is comatose and has a palpable depressed skull fracture. 18 breaths per minute. He has gross hematuria and a pelvic fracture. You decide to
His face is swollen and ecchymotic. He has gurgling respirations and vomitus on his transfer this patient to a facility capable of providing a higher level of care. The facility
face and clothing. The most appropriate step after providing supplemental oxygen is 128 km (80 miles)away. Before transfer, you should first:
and elevating his jaw is to:
15
a. intubate the patient
 a. avoid hypotension 

b. perform diagnostic peritoneal lavage or FAST b. administer an osmotic diuretic 

c. insert a left chest tube
d. call the receiving hospital and speak to the surgeon on call c. aggressively treat systemic hypertension 

e. discuss the advisability of transfer with the patient's family d. reduce metabolic requirements of the brain
e. distinguish between intracranial hematoma and cerebral edema
Hemorrhage of 20% of the patient's blood volume is associated usually with:

a. oliguria A 33-year-old woman is involved in a head-on motor vehicle crash. It took 30 minutes
b. confusion to extricate her from the car. Upon arrival in the emergency department, her heart rate
c. hypotension is 120 beats per
 minute, BP is 90/70 mm Hg, respiratory rate is 16 breaths per
d. tachycardia minute, and her GCS score is 15. Examination reveals bilaterally equal breath
e. blood transfusion requirement sounds, anterior chest wall ecchymosis, and distended neck veins. Her abdomen is
flat, soft, and not tender. Her pelvis is stable. Palpable distal pulses are found in all 4
Which one of the following statements concerning intraosseous infusion is TRUE?
extremities. Of the following, the most likely diagnosis is:

a. Only crystalloid solutions may be safely infused through the needle.
 a. hemorrhagic shock
b. Aspiration of bone marrow confirms appropriate positioning of the needle. b. cardiac tamponade
c. Intraosseous infusion is the preferred route for volume resuscitation in small c. massive hemothorax
children. d. tension pneumothorax
d. Intraosseous infusion may be utilized indefinitely.
 e. diaphragmatic rupture

e. Swelling in the soft tissues around the intraosseous site is not a reason to
A hemodynamically normal 10-year-old girl is admitted to the Pediatric Intensive Care
discontinue infusion.
Unit (PICU) for observation after a Grade III (moderately severe) splenic injury has
been confirmed by computed tomography (CT). Which of the following mandates
A young woman sustains a severe head injury as the result of a motor vehicle crash. prompt celiotomy (laparotomy)?
In the emergency department, her GCS is 6. Her blood pressure is 140/90 mm Hg
and her heart rate is 80 beats per minute. She is intubated and mechanically a. a serum amylase of 200

ventilated. Her pupils are 3 mm in size and equally reactive to light. There is no other b. a leukocyte count of 14,000

apparent injury. The most important principle to follow in the early management of her
head injury is to: c. extraperitoneal bladder rupture


16
d. free intraperitoneal air demonstrated on follow-up CT
 d. The legs are proportionally larger in infants than in adults.
e. The arms are proportionally larger in infants than in adults.
e. a fall in the hemoglobin level from 12 g/dL to 8 g/dL over 24 hours
A healthy young male in a motor vehicle crash is brought to the emergency
A.
 40-year-old woman restrained driver is transported to the emergency department department with a blood pressure of 84/60, pulse 123, GCS 10. The patient moans
in full spinal mmobilization. She is hemodynamically normal and found to be when his pelvis is palpated. After initiating fluid resuscitation, the next step in
paraplegic at the level of T10. . Neurologic examination also determines that there is management is:
loss of pain and temperature sensation with preservation of proprioception and
vibration. These findings are consistent with the diagnosis of: a. placement of a pelvic binder
b. transfer to a trauma center
a. central cord syndrome
c. pelvic x-ray

b. spinal shock syndrome
c. anterior cord syndrome d. insert urinary catheter
d. complete cord syndrome e. repeat examination of pelvis
e. Brown-Sequard's syndrome
Which one of the following situations requires Rh immunoglobulin administration to
A trauma patient presents to your emergency department with inspiratory stridor and an injure( woman?
a suspected c- spine injury. Oxygen saturation is 88% on high-flow oxygen via a
nonrebreathing mask. The most appropriate next step is to: a. negative pregnancy test, Rh negative, and has torso trauma

b. positive pregnancy test, Rh positive, and has torso trauma

a. apply cervical traction

c. positive pregnancy test, Rh negative, and has torso trauma

b. perform immediate tracheostomy

d. positive pregnancy test, Rh positive, and has an isolated wrist fracture
c. insert bilateral thoracostomy tubes
 e. positive pregnancy test, Rh negative, and has an isolated wrist fracture
d. maintain 100% oxygen and obtain immediate c-spine x-rays
e. maintain inline immobilization and establish a definitive airway
A 22-year-old female athlete is stabbed in her left chest at the third interspace in
When applying the Rule of Nines to infants: theanterior axillary line. On admission to the emergency department and 15 minutes
after the incident, she is awake and alert. Her heart rate is 100 beats per minute,
a. It is not reliable.
 blood pressure 80/60 mm Hg, and respiratory rate 20 breaths per minute. A chest x-
ray reveals a large left hemothorax. A left chest tube is placed with an immediate
b. The body is proportionally larger in infants than in adults.
return of1600 mL of blood. The next management step for this patient is:
c. The head is proportionally larger in infants than in adults.

17
a. perform a thoracoscopy 90 mm Hg systolic, heart rate is 140 beats per minute, and his respiratory rate is 36
b. perform an arch aortogram breaths per minute. The preferred route of venous access in this patient is:
c. insert a second left chest tube
d. prepare for an exploratory thoracotomy a. percutaneous femoral vein cannulation. 

e. perform a chest CT
b. cutdown on the saphenous vein at the ankle. 

A 6-year-old boy walking across the street is struck by the front bumper of a sports
c. intraosseous catheter placement in the proximal tibia. 

utility vehicle traveling at 32 kph (20 mph). Which one of the following statements is
TRUE about this patient? d. percutaneous peripheral veins in the upper extremities. 


e. central venous access via the subclavian or internal jugular vein. 



a. A flail chest is probable.

b. A symptomatic cardiac contusion is expected.
 Which one of the following physical findings suggests a cause of hypotension other
c. A pulmonary contusion may be present in the absence of rib fractures. than spinal cord injury?
d. Transection of the thoracic aorta is more likely than in an adult patient.

a. priapism. 

e. Rib fractures are commonly found in children with this mechanism of injury.
b. bradycardia. 

ATLS Practice Test 1
c. diaphragmatic breathing. 

1. Which one of the following is the recommended method for initially treating
d. presence of deep tendon reflexes. 

frostbite?
e. ability to flex forearms but inability to extend them. 

a. vasodilators 

b. anticoagulants 
 A young man sustains a gunshot wound to the abdomen and is brought promptly to
the emergency department by prehospital personnel. His skin is cool and diaphoretic,
c. warm (40°C) water 
 and he is confused. His pulse is thready and his femoral pulse is only weakly palpable.
d. padding and elevation 
 The definitive treatment in managing this patient is to:

e. application of heat from a hair dryer


a. administer O-negative blood. 

6-year-old boy is struck by an automobile and brought to the emergency department. b. apply external warming devices. 

He is lethargic, but withdraws purposefully from painful stimuli. His blood pressure is
18
c. control internal hemorrhage operatively. 
 d. perform diagnostic peritoneal lavage or FAST. 

d. apply a pneumatic antishock garment (PASG). 
 e. perform a urethrogram and cystogram. 

e. infuse large volumes of intravenous crystalloid solution. 

7. A
 42-year-old man, injured in a motor vehicle crash, suffers a closed head injury,
5. Regarding shock in the child, which of the following is FALSE? multiple palpable left rib fractures, and bilateral femur fractures. He is intubated
orotracheally without difficulty. Initially, his ventilations are easily assisted with a
a. Vital signs are age-related. 
 bag-mask device. It becomes more difficult to ventilate the patient over the next 5
minutes, and his hemoglobin oxygen saturation level decreases from 98% to 89% .
b. Children have greater physiologic reserves than do adults. 

The most appropriate next step is to:
c. Tachycardia is the primary physiologic response to hypovolemia. 

d. The absolute volume of blood loss required to produce shock is the same as a. obtain a chest x-ray. 


in 
 adults. 
 b. decrease the tidal volume. 



e. An initial fluid bolus for resuscitation should approximate 20 mL/kg of Ringer's c. decrease PEEP . 


 lactate. 

d. increase the rate of assisted ventilations. 


6. A
 33-year-old man is struck by a car traveling at 56 kph (35 mph). He has obvious e. perform needle decompression of the left chest. 

fractures of the left tibia near the knee, pain in the pelvic area, and severe dyspnea.
His heart rate is 182 beats per minute, and his respiratory rate is 48 breaths per 8. A young man sustains a rifle wound to the mid-abdomen. He is brought promptly
minute with no breath sounds heard in the left chest. A tension pneumothorax is to the emergency department by prehospital personnel. His skin is cool and
relieved by immediate needle decompression and tube thoracostomy. Subsequently, diaphoretic, and his systolic blood pressure is 58 mm Hg. Warmed crystalloid fluids
his heart rate decreases to 144 beats per minute, his respiratory rate decreases to 36 are initiated without improvement in his vital signs. The next, most appropriate, step
breaths per minute, and his blood pressure is 81/53 mm Hg. Warmed Ringer's lactate is to perform:
is administered intravenously. The next priority should be to:
a. a laparotomy. 

a. perform external fixation of the pelvis. 

b. an abdominal CT scan. 

b. obtain abdominal and pelvic CT scans. 

c. diagnostic laparoscopy. 

c. perform arterial embolization of the pelvic vessels. 

d. abdominal ultrasonography. 


19
e. a diagnostic peritoneal lavage. 
 11. A
 30-year-old man sustains a severely comminuted, open, distal right femur
fracture in a motorcycle crash. The wound is actively bleeding. Normal sensation is
9. The primary indication for transferring a patient to a higher level trauma center is: present over the lateral aspect of the foot but decreased over the medial foot and
great toe. Normal motion of the foot is observed. Dorsalis pedis and posterior tibial
a. unavailability of a surgeon or operating room staff. 
 pulses are easily palpable on the left, but heard only by Doppler on the right.
Immediate efforts to improve circulation to the injured extremity should involve:
b. multiple system injuries, including severe head injury. 

c. resource limitations as determined by the transferring doctor. 
 a. immediate angiography. 


d. resource limitations as determined by the hospital administration. 
 b. tamponade of the wound with a pressure dressing. 

c. wound exploration and removal of bony fragments.
e. widened mediastinum on chest x-ray following blunt thoracic trauma. 

d. realignment of the fracture segments with a traction splint. 


10. A
 42-year-old man is trapped from the waist down beneath his overturned e. fasciotomy of all four compartments in the lower extremity. 


tractor for several hours before medical assistance arrives. He is awake and alert until
just before arriving in the emergency department. He is now unconscious and 12. An 18-year-old, unhelmeted motorcyclist is brought by ambulance to the
responds only to painful stimuli by moaning. His pupils are 3 mm in diameter and emergency department following a crash. He had decreased level of consciousness
symmetrically reactive to light. Prehospital personnel indicate that they have not seen at the scene, but then was alert and conversational during transportation. Now his
the patient move either of his lower extremities. On examination in the emergency GCS is only 11. Which of the following statements is TRUE?
department, no movement of his lower extremities is detected, even in response to
painful stimuli. The most likely cause for this finding is: a. Cerebral perfusion is intact.

b. Intravascular volume status is normal.

a. an epidural hematoma. 

c. The patient is in a postictal state.

b. a pelvic fracture. 

d. Intra-abdominal visceral injuries are unlikely.

c. central cord syndrome. 

e. The patient probably has an acute epidural hematoma.
d. intracerebral hemorrhage. 

e. bilateral compartment syndrome. 
 13. During an altercation, a 36-year-old man sustains a gunshot wound above
the nipple line 
 on the right, with an exit wound posteriorly above the scapula on

20
the right. He is 
 transported by ambulance to a community hospital. He is a single bullet wound. He is breathing and has a thready pulse. However, he is
unconscious and has no detectable blood pressure. Optimal immediate management
endotracheally intubated, 
 closed tube thoracostomy is performed, and 2 liters of is to:
Ringer's lactate solution are 
 infused via 2 large-caliber IVs. His blood pressure now
a. perform diagnostic peritoneal lavage. 

is 60/0 mm Hg, heart rate is 160 
 beats per minute, and respiratory rate is 14 breaths
b. initiate infusion of packed red blood cells. 

per minute (ventilated with 100% 0 ). 
 The most appropriate next step in managing
c. insert a nasogastric tube and urinary catheter. 

this patient is:
d. transfer the patient to the operating room, while initiating fluid therapy. 

a. laparotomy. 
 e. initiate fluid therapy to return his blood pressure to normotensive 

b. diagnostic peritoneal lavage. 

c. arterial blood gas determination. 
 16. A
 teen-aged bicycle rider is hit by a truck traveling at high speed. In the
emergency department, she is actively bleeding from open fractures of her
d. administer packed red blood cells. 

legs, and has abrasions on her chest and abdominal wall. Her blood pressure
e. chest x-ray to confirm tube placement. 
 is 80/50 mm Hg, heart rate is 140 beats per minute, respiratory rate is 8
breaths per minute, and GCS score is 6. The first step in managing this
14. Absence of breath sounds and dullness to percussion over the left patient is to:
hemithorax are findings best explained by:
a. obtain a lateral cervical spine x-ray. 

a. left hemothorax. 
 b. insert a central venous pressure line. 

b. cardiac contusion 
 c. administer 2 liters of crystalloid solution. 

c. left simple pneumothorax 
 d. perform endotracheal intubation and ventilation. 

d. left diaphragmatic rupture 
 e. apply a pneumatic antishock garment (PASG) and inflate the leg

e. right tension pneumothorax. 
 compartments. 


15. A 23-year-old man is brought immediately to the emergency department from the 17. An 8-year-old boy falls 4.5 meters (15 feet) from a tree and is brought to the
hospital's parking lot where he was shot in the lower abdomen. Examination reveals emergency department by his family. His vital signs are normal, but he

21
complains of left upper quadrant pain. An abdominal CT scan reveals a heart rate of 90 beats per minute, and respiratory rate of 22 breaths per minute. His
moderately severe laceration of the spleen. The receiving institution does not respirations are sonorous and deep. His GCS score is 6. Immobilization of the entire
have 24-hour-a-day operating room capabilities. The most appropriate patient may include the use of all the following EXCEPT:
management of this patient would be to:
a. air splints. 

a. type and crossmatch for blood. 
 b. bolstering devices. 

b. request consultation of a pediatrician. 
 c. a long spine board. 

c. transfer the patient to a trauma center. 
 d. a scoop-style stretcher. 

d. admit the patient to the intensive care unit. 
 e. a semi-rigid cervical collar. 

e. prepare the patient for surgery the next day. 


18. Which of the following statements regarding injury to the central nervous
system in children is TRUE ? 20. Twenty-seven patients are seriously injured in an aircraft accident at a local
airport. The basic principle of triage should be to:
a. Children suffer spinal cord injury without x-ray abnormality more commonly
than adults. 
 a. treat the most severely injured patients first. 

b. An infant with a traumatic brain injury may become hypotensive from cerebral b. establish a field triage area directed by a doctor. 

edema. 
 c. rapidly transport all patients to the nearest appropriate hospital. 

c. Initial therapy for the child with traumatic brain injury includes the
d. treat the greatest number of patients in the shortest period of time. 

administration of methylprednisolone intravenously.
d. Children have more focal mass lesions as a result of traumatic brain injury e. produce the greatest number of survivors based on available resources. 

when compared to adults . 

e. Young children are less tolerant of expanding intracranial mass lesions than 21. An electrician is electrocuted by a downed power line after a thunderstorm.
He apparently made contact with the wire at the level of the right mid thigh.
adults. 

In the emergency department, his vital signs are normal and no dysrhythmia
is noted on ECG. On examination, there is an exit wound on the bottom of
19. A 17-year-old helmeted motorcyclist is struck broadside by an automobile at an the right foot. His urine is positive for blood by dipstick but no RBCs are seen
intersection. He is unconscious at the scene with a blood pressure of 140/90 mm Hg, microscopically. Initial management should include:
22
a. immediate angiography. 
 e. clinical evidence of inadequate organ perfusion must be present. 

b. aggressive fluid infusion. 

24. A
 32-year-old man is brought to the hospital unconscious with severe facial
c. intravenous pyelography. 

injuries and noisy respirations after an automobile collision. In the emergency
d. debridement of necrotic muscle. 
 department, he has no apparent injury to the anterior aspect of his neck. He suddenly
e. admission to the intensive care unit for observation. becomes apneic, and attempted ventilation with a face mask is unsuccessful.
Examination of his mouth reveals a large hematoma of the pharynx with loss of normal
22. A young woman sustains a severe head injury as the result of a motor anatomic landmarks. Initial management of his airway should consist of:
vehicular crash. In the emergency department, her GCS score is 6. Her blood
pressure is 140/90 mm Hg and her heart rate is 80 beats per minute. She is a. inserting an oropharyngeal airway. 

intubated and is being mechanically ventilated. Her pupils are 3 mm in size
b. inserting a nasopharyngeal airway. 

and equally reactive to light. There is no other apparent injury. The most
important principle to follow in the early management of her head injury is to: c. performing a surgical cricothyroidotomy. 

d. performing fiberoptic-guided nasotracheal intubation. 

a. administer an osmotic diuretic. 

e. performing orotracheal intubation after obtaining a lateral c-spine x-ray. 

b. prevent secondary brain injury. 

c. aggressively treat systemic hypertension. 
 25. A
 25-year-old woman is brought to the emergency department after a motor
d. reduce metabolic requirements of the brain. 
 vehicle crash. She was initially lucid at the scene and then developed a dilated pupil
and contralateral extremity weakness. In the emergency department, she is
e. distinguish between intracranial hematoma and cerebral edema. 

unconscious and has a GCS score of 6. The initial management step for this patient
should be to:
23. To establish a diagnosis of shock,
a. obtain a CT scan of the head. 

a. systolic blood pressure must be below 90 mm Hg. 
 b. administer decadron 20 mg IV.
b. the presence of a closed head injury should be excluded 
 c. perform endotracheal intubation.
d. administer mannitol 1 g/kg IV . 

c. acidosis should be present by arterial blood gas analysis 

e. perform an emergency bone flap craniotomy on the side of the dilated pupil.
d. the patient must fail to respond to intravenous fluid infusion. 


23
26. A contraindication to nasogastric intubation is the presence of a: a. Pneumothorax 

a. gastric perforation. b. Pneumomediastinum 

b. diaphragmatic rupture. 
 c. Massive hemothorax 

c. open depressed skull fracture. 
 d. Diaphragmatic rupture 

d. fracture of the cervical spine. 
 e. Subcutaneous emphysema 

e. fracture of the cribriform plate. 

29. Cardiac tamponade after trauma:
27. An 8-year-old girl is an unrestrained passenger in a vehicle struck from behind. In
the emergency department, her blood pressure is 80/60 mm Hg, heart rate is 80 a. is seldom life-threatening. 

beats per minute, and respiratory rate is 16 breaths per minute. Her GCS score is
14. She complains that her legs feel "funny and won't move right." However, her b. can be excluded by an upright, AP chest x-ray. 

spine x-rays do not show a fracture or dislocation. A spinal cord injury in this child: c. can be confused with a tension pneumothorax. 

d. causes a fall in systolic pressure of > 15 mm Hg with expiration. 

a. is most likely a central cord syndrome. 

e. most commonly occurs after blunt injury to the anterior chest wall. 

b. must be diagnosed by magnetic resonance imaging. 

c. can be excluded by obtaining a CT of the entire spine. 
 30. A 22-year-old man is brought to the hospital after crashing his motorcycle into a
d. may exist in the absence of objective findings on x-ray studies. 
 telephone pole. He is unconscious and in profound shock. He has no open
wounds or obvious fractures. The cause of his shock is MOST LIKELY cause d
e. is unlikely because of the incomplete calcification of the vertebral bodies. 

by: 

a. a subdural hematoma. 

b. an epidural hematoma. 

c. a transected lumbar spinal cord. 

28. Immediate chest tube insertion is indicated for which of the following d. a basilar skull fracture. 

conditions?
e. hemorrhage into the chest or abdomen.

24
b. patient with an inhalation injury. 

c. 6-year-old child with a pelvic fracture. 


31. Which one of the following statements is FALSE concerning Rh d. patient with a severe cardiac contusion. 

isoimmunizationin the pregnant trauma patient? e. 24-year-old man with a massive hemothorax. 


a. It occurs in blunt or penetrating abdominal trauma. 



b. Minor degrees of fetomaternal hemorrhage produce it. 

34. A cross-table latera; x-ray of the cervical spine :
c. A negative Kleihauer-Betke test excludes Rh isoimmunization. 
 a. Must precede endotracheal intubation
d. This is not a problem in the traumatized Rh-positive pregnant patient. 
 b. Excludes serious cervical spine injury
c. is an essential part of the primary survey.
e. initiation of Rh immunoglobulin therapy does not require proof of fetomaternal
d. is not necessary for unconscious patients with penetrating cervical injuries.

 hemorrhage. 
 e. is unacceptable unless 7 cervical vertebrae and the C-7 to T-1 relationship are
visualized.
32. All of the following signs on the chest x-ray of a blunt injury victim may
suggest aortic rupture E XCEPT: 35. A 24-year-old man sustains multiple fractured ribs bilaterally as a result of being
crushed in a press at a plywood factory. Examination in the emergency department
reveals a flail segment of the patient's thorax. Primary resuscitation includes
a. mediastinal emphysema. 

high-flow oxygen administration via a nonrebreathing mask, and initiation of
b. presence of a "pleural cap." 
 Ringer's lactate solution. The patient exhibits progressive confusion, cyanosis, and
tachypnea. Management at this time should consist of:
c. obliteration of the aortic knob. 

d. deviation of the trachea to the right. 
 a. intravenous sedation. 

e. depression of the left mainstem bronchus 
 b. external stabilization of the chest wall. 

c. increasing the FIO 2 in the inspired gas. 

33. Early central venous pressure monitoring during fluid resuscitation in the
emergency department has the greatest utility in a: d. intercostal nerve blocks for pain relief. 


e. endotracheal intubation and mechanical ventilation. 



a. patient with a splenic laceration. 

25
36. Which one of the following statements regarding patients with thoracic spine c. carotid pulse assessment.
injuries is TRUE? d. management of blood loss. 


e. determination of associated Injuries. 



a. Log-rolling may be destabilizing to fractures from T12 to L1. 

b. Adequate immobilization can be accomplished with the scoop stretcher. 
 39. The driver of a single car crash is orotracheally intubated in the field by prehospital
c. Spinal cord injury below T-10 usually spares bowel and bladder function. 
 personnel after they identify a closed head injury and determine that the patient is
unable to protect his airway. In the emergency department, the patient demonstrates
d. Hyperflexion fractures in the upper thoracic spine are inherently unstable. 
 decorticate posturing bilaterally. He is being ventilated with a bag-valve device, but
e. These patients rarely present with neurogenic shock in association with cord his breath sounds are absent in the left hemithorax. His blood pressure is 160/88 mm

 injury. 
 Hg, heart rate is 70 beats per minute, and the pulse oximeter displays a hemoglobin
oxygen saturation of 96%. The next step in assessing and managing this patient
should be to:
37. During resuscitation, which one of the following is the most reliable as a guide to
volume replacement?
a. determine the arterial blood gases. 


a. heart rate 
 b. obtain a lateral cervical spine x-ray. 


b. hematocrit 
 c. assess placement of the endotracheal tube. 


c. blood pressure 
 d. perform needle decompression of the left chest. 


d. urinary output 
 e. insert a thoracostomy tube in the left hemithorax. 


e. jugular venous pressure 



40. The response to catecholamines in an injured, hypovolemic pregnant woman can
be expected to result in:
38. A
 24-year-old woman passenger in an automobile strikes the wind screen with
her face during a head-on collision. In the emergency department, she is talking and a. placental abruption. 

has marked facial edema and crepitus. The highest priority should be given to: b. fetal hypoxia and distress. 


a. lateral c-spine x-ray. 
 c. fetal/maternal dysrhythmia. 


b. upper airway protection. 
 d. improved uterine blood flow. 


26
e. increased maternal renal blood flow. 
 c. aggressively treat systemic hypertension. 

d. reduce metabolic requirements of the brain. 

ATLS Practice Test 2
e. distinguish between intracranial hematoma and cerebral edema. 

1. A 22-year-old man sustains a gunshot wound to the left chest and is transported to
a small community hospital at which surgical capabilities are not available. In the 3. A
 6-year-old boy walking across the street is struck by the front bumper of a
emergency department, a chest tube is inserted and 700 mL of blood is evacuated.
sports utility vehicle traveling at 32 kph ( 20 mph ). Which one of the following
The trauma center accepts the patient in transfer. Just before the patient is placed in
statements is TRUE ?
the ambulance for transfer, his blood pressure decreases to 80/68 mm Hg and his
heart rate increases to 136 beats per minute. The next step should be to:
a. A flail chest is probable. 

a. clamp the chest tube. 
 b. A symptomatic cardiac contusion is expected. 

b. cancel the patient's transfer. 
 c. A pulmonary contusion may be present in the absence of rib fractures. 

c. perform an emergency department thoracotomy. 
 d. Transection of the thoracic aorta is more likely than in an adult patient. 

d. repeat the primary survey and proceed with transfer. 
 e. Rib fractures are commonly found in children with this mechanism of injury.

e. delay the transfer until the referring doctor can contact a thoracic surgeon.

 4. A
 39-year-old man is admitted to the emergency department after an automobile
collision. He is cyanotic, has insufficient respiratory effort, and has a GCS Score of 6.
His full beard makes it difficult to fit the oxygen facemask to his face. The most
2. A young woman sustains a severe head injury as the result of a motor vehicle appropriate next step is to:
crash. In the emergency department, her GCS is 6. Her blood pressure is 140/90 mm
Hg and her heart rate is 80 beats per minute. She is intubated and mechanically a. perform a surgical cricothyroidotomy. 

ventilated. Her pupils are 3 mm in size and equally reactive to light. There is no other
apparent injury. The most important principle to follow in the early management of her b. attempt nasotracheal intubation. 

head injury is to: c. ventilate him with a bag-mask device until c-spine injury can be excluded.
d. attempt orotracheal intubation using 2 people and inline stabilization of the
a. avoid hypotension. 
 
 cervical spine. 

b. administer an osmotic diuretic. 
 e. ventilate the patient with a bag-mask device until his beard can be shaved for

27

 better mask fit. 
 at the hospital, but increases to 110/70 mm Hg with the administration of 2 liters
of intravenous fluid. His heart rate remains 120 beats per minute. Computed
tomography shows an aortic injury and splenic laceration with free abdominal fluid.
5. A patient is brought to the emergency department 20 minutes after a motor vehicle
His blood pressure falls to 70 mm Hg after CT. The next step is:
crash. He is conscious and there is no obvious external trauma. He arrives at the
hospital completely immobilized on a long spine board. His blood pressure is 60/40
mm Hg and his heart rate is 70 beats per minute. His skin is warm. Which one of a. contrast angiography. 


the following 
 statements is TRUE ? b. transfer to a higher level trauma center. 



c. exploratory laparotomy. 

a. Vasoactive medications have no role in this patient's management.

d. transfuse packed red blood cells. 

b. The hypotension should be managed with volume resuscitation alone.
e. transesophageal echocardiography. 

c. Flexion and extension views of the c-spine should be performed early.

d. Occult abdominal visceral injuries can be excluded as a cause of
8. Which one of the following statements regarding abdominal trauma in the pregnant
hypotension.
e. Flaccidity of the lower extremities and loss of deep tendon reflexes are is TRUE? 

expected. 

a. The fetus is in jeopardy only with major abdominal trauma.

6. The following are contraindications for tetanus toxoid administration: b. Leakage of amniotic fluid is an indication for hospital admission.

c. Indications for peritoneal lavage are different from those in the nonpregnant
a. History of neurological reaction or severe hypersensitivity to the product. 

patient.

b. Local side effects. 
 d. Penetration of an abdominal hollow viscus is more common in late than in
c. Muscular spasms. 
 early pregnancy.

d. Pregnancy. 
 e. The secondary survey follows a different pattern from that of the nonpregnant
patient.
e. All of the above. 

9. All of the following are indicators of inhalation injury, EXCEPT:
7. After being involved in a motor vehicle crash, a 25-year-old man is brought to a
hospital with a general surgeon on duty. He has a GCS of 13 and complains of a. singeing of the eyebrows and nasal vibrissae. 

abdominal pain. His blood pressure was 80 mm Hg systolic by palpation on arrival
28
b. carboxyhemoglobin level >4%. 
 a. request a CT scan. 

c. carbon deposits in the mouth or nose, and carbonaceous sputum. 
 b. insert a gastric tube. 

d. hoarseness. 
 c. suction the oropharynx. 


e. face or neck burns. 
 d. obtain a lateral cervical spine x-ray. 


e. ventilate the patient with a bag-mask. 


12. A
 64-year-old man, involved in a high-speed car crash, is resuscitated initially


10. A
 32-year-old man's right leg is trapped beneath his overturned car for nearly 2
in a small hospital with limited resources. He has a closed head injury with a GCS
hours before he is extricated. On arrival in the emergency department, his right
Score of 13. He has a widened mediastinum on chest x-ray with fractures of left
lower extremity is cool, mottled, insensate, and motionless. Despite normal vital
ribs 2 through 4, but no pneumothorax. After infusing 2 liters of crystalloid solution,
signs, pulses cannot be palpated inferior to the femoral artery, and the muscles of
his blood pressure is 100/74 mm Hg, heart rate is 110 beats per minute, and
the lower extremity are firm and hard. During the initial management of this patient,
respiratory rate is 18 breaths per minute. He has gross hematuria and a pelvic
which of the following is most likely to improve the chances for limb salvage?
fracture. You decide to transfer this patient to a facility capable of providing a
higher level of care. The facility is 128 km (80 miles) away. Before transfer, you
a. Applying skeletal traction. 

should first:
b. Administering anticoagulant drugs. 

a. intubate the patient. 

c. Administering thrombolytic therapy. 

b. perform diagnostic peritoneal lavage. 

d. Perform right lower extremity fasciotomy. 

c. apply the pneumatic antishock garment. 

e. Immediately transferring the patient to a trauma center. 

d. call the receiving hospital and speak to the surgeon on call. 


11. A
 patient arrives in the emergency department after being beaten about the e. discuss the advisability of transfer with the patient's family. 

head and face with a wooden club. He is comatose and has a palpable depressed
skull fracture. His face is swollen and ecchymotic. He has gurgling respirations 13. During the third trimester of pregnancy, all of the following changes occur
and vomitus on his face and clothing. The most appropriate step after providing normally, EXCEPT a:
supplemental oxygen and elevating his jaw is to:
a. decrease in PaCO2 . 


29
b. decrease in leukocyte count. d. reducing the volume of crystalloid required for resuscitation. 

c. reduced gastric emptying rate.
d. diminished residual lung volume. e. increasing the volume of blood loss to produce maternal hypotension. 

e. diminished pelvic ligament tension.
17. The first maneuver to improve oxygenation after chest injury is:
14. In managing the head-injured patient, the most important initial step is to:
a. secure the airway. a. intubate the patient. 

b. obtain c-spine film.
b. assess arterial blood gases. 

c. support the circulation.
d. control scalp hemorrhage. c. administer supplemental oxygen. 

e. determine the GCS Score.
d. ascertain the need for a chest tube. 

15. A
 previously healthy, 70-kg (154-pound) man suffers an estimated acute blood e. obtain a chest x-ray. 

loss of 2 liters. Which one of the following statements applies to this patient?
18. A 25-year-old man, injured in a motor vehicular crash, is admitted to the
a. His pulse pressure will be widened. 
 emergency department. His pupils react sluggishly and his eyes open to painful
stimuli only. He does not follow commands, but he does moan periodically. His
b. His urinary output will be at the lower limits of normal. 
 right arm is deformed and does not respond to painful stimulus; however, his left
c. He will have tachycardia, but no change in his systolic blood pressure. 
 hand reaches purposefully toward the painful stimulus. Both legs are stiffly
extended. His GCS score is:
d. His systolic blood pressure will be decreased with a narrowed pulse pressure.
e. His systolic blood pressure will be maintained with an elevated diastolic
a. 7 

pressure. 

b. 8 

16. The physiologic hypervolemia of pregnancy has clinical significance in the c. 9 

management of the severely injured, gravid woman by:
d. 10 


a. reducing the need for blood transfusion. 
 e. 11 


b. increasing the risk of pulmonary edema. 



19. A
 20-year-old woman, at 32 weeks gestation, is stabbed in the upper right
c. complicating the management of closed head injury. 

chest. In the emergency department, her blood pressure is 80/60 mm Hg.
30
She is gasping for breath, extremely anxious, and yelling for help. Breath d. maintain 100% oxygen and obtain immediate c-spine x-rays. 

sounds are diminished in the right chest. The most appropriate first step is to:
e. maintain inline immobilization and establish a definitive airway. 

a. perform tracheal intubation. 

22. When applying the Rule of Nines to infants,
b. insert an oropharyngeal airway. 

c. perform needle decompression of the right chest. 
 a. it is not reliable. 

d. manually displace the gravid uterus to the left side of the abdomen. 
 b. the body is proportionally larger in infants than in adults. 

e. initiate 2 large-caliber peripheral IV lines and crystalloid infusion. 
 c. the head is proportionally larger in infants than in adults. 

d. the legs are proportionally larger in infants than in adults. 

20. Which one of the following findings in an adult should prompt immediate
management during the primary survey? e. the arms are proportionally larger in infants than in adults. 


a. Distended abdomen. 
 26. A
 7-year-old boy is brought to the emergency department by his parents several
b. Glasgow Coma Scale Score of 11. 
 minutes after he fell through a window. He is bleeding profusely from a 6-cm
wound of his medial right thigh. Immediate management of the wound should
c. Temperature of 36.5°C (97.8°F). 
 consist of:
d. Heart rate of 120 beats per minute. 

a. application of a tourniquet. 

e. Respiratory rate of 40 breaths per minute. 

b. direct pressure on the wound. 


21. A
 trauma patient presents to your emergency department with inspiratory c. packing the wound with gauze. 

stridor and a suspected c-spine injury. Oxygen saturation is 88% on high-flow d. direct pressure on the femoral artery at the groin. 

oxygen via a nonrebreathing mask. The most appropriate next step is to:
e. debridement of devitalized tissue. 


a. apply cervical traction. 



27. For the patient with severe traumatic brain injury, profound hypocarbia should be
b. perform immediate tracheostomy. 
 avoided to prevent:

c. insert bilateral thoracostomy tubes. 



31
a. respiratory alkalosis. 
 e. Inhalation injury 

b. metabolic acidosis. 

25. Systolic blood pressure starts to decrease in which class of hemorrhage?
c. cerebral vasoconstriction with diminished perfusion. 

d. neurogenic pulmonary edema. 
 a. Class 0 

e. shift of the oxyhemoglobin dissociation curve. 
 b. Class 1 

c. Class 2 

23. A 60-year-old man sustains a stab wound to the right posterior flank. Witnesses
state the weapon was a small knife. His heart rate is 90 beats per minute, blood d. Class 3 

pressure is 128/72 mm Hg, and respiratory rate is 24 breaths per minute. The most e. Class 4 

appropriate action to take at this time is to:

a. perform a colonoscopy. 
 26. A
 7-year-old boy is brought to the emergency department by his parents several
minutes after he fell through a window. He is bleeding profusely from a 6-cm wound
b. perform a barium enema. 

of his medial right thigh. Immediate management of the wound should consist of:
c. perform an intravenous pyelogram. 

a. application of a tourniquet. 

d. perform serial physical examinations. 

b. direct pressure on the wound. 

e. suture repair the wound and outpatient follow up. 

c. packing the wound with gauze. 

24. The following are criteria for transfer to a burn center, EXCEPT for: d. direct pressure on the femoral artery at the groin. 


e. debridement of devitalized tissue. 



a. Partial-thickness and full-thickness burns on greater than 10% of the BSA 

b. Any full-thickness burn 
 27. For the patient with severe traumatic brain injury, profound hypocarbia should be
c. Partial-thickness and full-thickness burns involving the face, hands, feet, avoided to prevent:
genitalia, perineum, and skin overlying major joints 

a. respiratory alkalosis. 

d. Elevated central venous pressure 

b. metabolic acidosis. 


32
c. cerebral vasoconstriction with diminished perfusion. 
 d. Free intraperitoneal air demonstrated on follow-up CT. 

d. neurogenic pulmonary edema. 
 e. A fall in the hemoglobin level from 12 g/dL to 8 g/dL over 24 hours. 

e. shift of the oxyhemoglobin dissociation curve. 

30. A
 40-year-old woman restrained driver is transported to the emergency
28. A 33-year-old woman is involved in a head-on motor vehicle crash. It took 30 department in full spinal immobilization. She is hemodynamically normal and found
minutes to extricate her from the car. Upon arrival in the emergency department, her to be paraplegic at the level of T10. Neurologic examination also determines that
heart rate is 120 beats per minute, BP is 90/70 mm Hg, respiratory rate is 16 breaths there is loss of pain and temperature sensation with preservation of proprioception
per minute, and her GCS Score is 15. Examination reveals bilaterally equal breath and vibration. These findings are consistent with the diagnosis of:
sounds, anterior chest wall ecchymosis, and distended neck veins. Her abdomen is a. central cord syndrome. 

flat, soft, and not tender. Her pelvis is stable. Palpable distal pulses are found in all 4
extremities. Of the following, the most likely diagnosis is: b. spinal shock syndrome. 

c. anterior cord syndrome. 

a. hemorrhagic shock. 

d. complete cord syndrome. 

b. cardiac tamponade. 

e. Brown-Séquard syndrome. 

c. massive hemothorax. 

d. tension pneumothorax. 
 31. Hemorrhage of 20% of the patient's blood volume is associated usually with:

e. diaphragmatic rupture. 
 a. oliguria. 



b. confusion. 

29. A
 hemodynamically normal 10-year-old girl is admitted to the Pediatric c. hypotension. 

Intensive Care Unit (PICU) for observation after a Grade III (moderately severe)
d. tachycardia. 

splenic injury has been confirmed by computed tomography (CT). Which of the
following mandates prompt laparotomy? e. blood transfusion requirement. 

32. Which one of the following statements concerning intraosseous infusion is TRUE?
a. A serum amylase of 200. 

a. Only crystalloid solutions may be safely infused through the needle. 

b. A leukocyte count of 14,000. 

b. Aspiration of bone marrow confirms appropriate positioning of the needle. 

c. Extraperitoneal bladder rupture. 
 c. Intraosseous infusion is the preferred route for volume resuscitation in small

33
children. 
 a closed tube thoracostomy is performed for decreased left breath sounds with the
return of a small amount of blood and no air leak. After chest tube insertion, the
d. Intraosseous infusion may be utilized indefinitely. 
 most appropriate next step is:
e. Swelling in the soft tissues around the intraosseous site is not a reason to a. reexamine the chest. 

discontinue infusion. 

b. perform an aortogram. 


33. The most important, immediate step in the management of an open pneumothorax c. obtain a CT scan of the chest. 

is: d. obtain arterial blood gas analyses. 

a. endotracheal intubation. 

e. perform transesophageal echocardiography. 

b. operation to close the wound. 

c. placing a chest tube through the chest wound. 
 36. A construction worker falls two stories from a building and sustains bilateral
calcaneal fractures. In the emergency department, he is alert, vital signs are
d. placement of an occlusive dressing over the wound. 
 normal, and he is complaining of severe pain in both heels and his lower back.
e. initiation of 2 large-caliber IVs with crystalloid solution. 
 Lower extremity pulses are strong and there is no other deformity. The suspected
diagnosis is most likely to be confirmed by:

34. Which one of the following situations requires Rh immunoglobulin administration a. angiography. 

to an injured woman?
b. compartment pressures. 

a. Negative pregnancy test, Rh negative, and torso trauma.

b. Positive pregnancy test, Rh positive, and has torso trauma. c. retrograde urethrogram. 

c. Positive pregnancy test, Rh negative, and has torso trauma.
 d. doppler-ultrasound studies. 

d. Positive pregnancy test, Rh positive, and has an isolated wrist fracture. e. complete spine x-ray series. 

e. Positive pregnancy test, Rh negative, and has an isolated wrist fracture.

35. A 22-year-old man is hypotensive and tachycardic after a shotgun wound to the 37. A
 22-year-old female athlete is stabbed in her left chest at the third interspace
left shoulder. His blood pressure is initially 80/40 mm Hg. After 2 liters of crystalloid in the anterior axillary line. On admission to the emergency department and 15
solution his blood pressure increases to 122/84 mm Hg. His heart rate is now 100 minutes after the incident, she is awake and alert. Her heart rate is 100 beats
beats per minute and his respiratory rate is 28 breaths per minute. His breath per minute, blood pressure 80/60 mm Hg, and respiratory rate 20 breaths per
sounds are decreased in the left hemithorax, and after initial IV fluid resuscitation, minute. A chest x-ray reveals a large left hemothorax. A left chest tube is placed

34
with an immediate return of 1600 mL of blood. The next management step for d. It comprises a visual assessment of the distance from the tongue base to the
this patient is: roof of the mouth, and therefore the amount of space in which there is to
work. 

a. perform a thoracoscopy. 

e. A poor Mallampati score is associated with a higher incidence of obstructive
b. perform an arch aortogram. 
 sleep apnea. 

c. insert a second left chest tube. 

40. A 23-year-old man sustains three stab wounds to the upper right chest during an
d. prepare for an exploratory thoracotomy. 

altercation and is brought by ambulance to a hospital that has full surgical
e. perform a chest CT. capabilities. His wounds are all above the nipple. He is endotracheally intubated,
closed tube thoracostomy is performed, and 2 liters of crystalloid solution are
38. A
 56-year-old man is thrown violently against the steering wheel of his truck infused through 2 large-caliber IVs. His blood pressure now is 60/0 mm Hg, heart
during a motor vehicle crash. On arrival in the emergency department he is rate is 160 beats per minute, and respiratory rate is 14 breaths per minute
diaphoretic and complaining of chest pain. His blood pressure is 60/40 mm Hg and (ventilated with 100% O ). 1500 mL of blood has drained from the right chest.
his respiratory rate is 40 breaths per minute. Which of the following best The most appropriate next step in managing this patient is to:
differentiates cardiac tamponade from tension pneumothorax as the cause of his
hypotension? a. perform FAST. 

a. Tachycardia. 
 b. obtain a CT of the chest. 

b. Pulse volume. 
 c. perform an angiography. 


c. Breath sounds. 
 d. urgently transfer the patient to the operating room. 


d. Pulse pressure. 
 e. immediately transfer the patient to a trauma center. 



e. Jugular venous pressure. 

ATLS Practice Test 3
39. All of the following are true of the Mallampati classification, EXCEPT:
1. Signs and symptoms of airway compromise include all of the following except:
a. Class IV is the easiest intubation, while Class I is the most difficult.. 

b. It helps assess for difficult intubations. 
 a. change in voice. 


c. It is part of the LEMON assessment. 
 b. stridor. 


35
c. decreased pulse pressure. 
 Cervical spine x-rays:
a. will show cervical spine injury in more than 20% of these patients. 

d. dyspnea and agitation. 

b. will exclude cervical spine injury if no abnormalities are found on the x-rays.
e. tachypnea. 
 c. are not needed if she is awake, alert, neurologically normal, and has no neck
pain 
 or midline tenderness. 

2. A
 29-year-old female arrives in the emergency department after being involved d. should be performed before addressing potential breathing or circulatory
in a motor vehicle crash. She is 30 weeks pregnant. She was restrained with a lap 
 problems. 

and shoulder belt, and an airbag deployed. Which one of the following statements
best describes the risk of injury? e. may show atlanto-occipital dislocation if the Power's ratio is < 1. 


a. The deployment of the airbag increases the risk of fetal loss. 
 5. The most specific test to evaluate for injuries of solid abdominal organs is:

b. The use of seatbelts is associated with increased risk of maternal death, 
 a. abdominal x-rays 

c. The mechanism of injury suggests the need for emergency caesarean
b. abdominal ultrasonography 

section due 
 to the risk of impending abruptio placentae. 

c. diagnostic peritoneal lavage 

d. The risk of premature fetal delivery and death is reduced by the use of
restraints. 
 d. frequent abdominal examinations 

e. The deployment of the airbag increases the risk of maternal abdominal injury. e. CT of abdomen and pelvis 


3. Cardiac tamponade: 6. A 40-year-old obese patient with a Glasgow Coma Scale score of 8 requires a CT
Scan. transfer to the scanner, you should:

a. is definitively managed by needle pericardiocentesis 

b. is most common with blunt thoracic trauma and anterior rib fractures 
 a. give more sedative drugs.

c. is easily diagnosed by discovery of Beck's triad in the emergency department b. insert a multi-lumen esophageal airway.
d. is indicated by Kussmaul breathing 
 c. insert a definitive airway.

e. requires surgical intervention 
 d. request a lateral cervical spine film.
e. insert a nasogastric tube.
4. A 14-year-old female is brought to the emergency department after falling from a
horse. She is immobilized on a long spine board with a hard collar and blocks.
36
7. A
 23-year-old construction worker is brought to the emergency department after a. multiple organ failure 

falling more than 9 meters (30 feet) from scaffolding. His vital signs are: heart rate b. decreased base deficit 

140, blood pressure 96/60 mm Hg, and respiratory rate 36. He is complaining
bitterly of lower abdominal and lower limb pain, and has obvious deformity of both c. acute glomerulonephritis 

lower legs with bilateral open tibial fractures. Which one of the following d. increased cellular adenosine triphosphate (A TP) production 

statements concerning this patient is true?

e. vasodilation 


a. Pelvic injury can be ruled out based on the mechanism of injury.



10. Hypertension following a head injury:
b. Blood loss from the lower limbs is the most likely cause of his hypotension.
c. X-rays of the chest and pelvis are important adjuncts in his initial assessment.
a. should be treated to reduce intracranial pressure 

d. Spinal cord injury is the most likely cause of his hypotension.

b. may indicate imminent herniation from critically high intracranial pressure
e. Aortic injury is the most likely cause of his tachycardia.
c. indicates pre-existing hypertension 

8. A 25-year-old female in the third trimester of pregnancy is brought to the emergency d. mandates prompt administration of mannitol 

department following a high-speed motor vehicle crash. She is conscious and
immobilized on a long spine board. Her respiratory rate is 24, heart rate is 120, e. should prompt burr hole drainage of potential subdural hematomas 

and blood pressure is 70/50. The laboratory results show a PaCO 2 of 40 mm Hg.
Which one of the following statements concerning this patient is true? 11. Initial treatment of frostbite injuries involves:

a. Fetal assessment should take priority. 
 a. application of dry heat. 



b. Log-rolling the patient to the right will decompress the vena cava. 
 b. rapid rewarming of the body part in circulating warm water. 

c. Rh-immunoglobulin therapy should be immediately administered. 
 c. debridement of hemorrhagic blisters. 

d. The patient likely has impending respiratory failure. 
 d. early amputation to prevent septic complications. 

e. Vasopressors should be given to the patient. 
 e. massage of the affected area. 


9. The most important consequence of inadequate organ perfusion is:

37
12. Which one of the following statements is true regarding a pregnant patient who a. administer heliox and racemic epinephrine 

presents following blunt trauma?
b. perform nasotracheal intubation 

a. Early gastric decompression is important. 
 c. perform surgical cricothyroidotomy 

b. A hemoglobin level of 10 g/dL (hematocrit 30) indicates recent blood loss. d. repeat orotracheal intubation 

c. The central venous pressure response to volume resuscitation is blunted in
e. perform needle cricothyroidotomy with jet insufflation 


 pregnant patients. 

d. A lap belt is the best form of restraint due to the size of the gravid uterus. 
 15. A 28-year-old male is brought to the emergency department. He was involved in
e. A PaCO 2 of 40 mmHg (5.3 kPa) provides reassurance about the adequacy a fight in which he was beaten with a wooden stick. His chest shows multiple
severe bruises. His airway is clear, respiratory rate is 22, heart rate is 126, and
of 
 respiratory function 

systolic blood pressure is 90 mm Hg. Which of the following should be performed
during the primary survey?
13. Which of the following signs is l east reliable for diagnosing esophageal
intubation? a. Glasgow Coma Score 

b. tetanus toxoid administration 

a. symmetrical chest wall movement 

c. cervical spine x-ray 

b. end-tidal CO 2 

d. blood alcohol-level 

c. bilateral breath sounds 

e. rectal exam 

d. oxygen saturation 

e. chest x-ray demonstrating the ETT tip positioned above the carina 
 16. Which one of the following injuries is addressed in the secondary survey?

14. A 6-month-old infant, being held in her mother's arms, is ejected on impact from a. forearm fracture 

a vehicle that is struck head-on by an oncoming car traveling at 64 kph (40 mph).
b. mid-thigh amputation 

The infant arrives in the emergency department with multiple facial injuries, is
lethargic, and is in severe respiratory distress. Respiratory support is not c. open fracture with bleeding 

effective using a bag-mask device, and her oxygen saturation is falling.
d. unstable pelvic fracture 

Repeated attempts at orotracheal intubation are unsuccessful. The most
appropriate procedure to perform next is: e. bilateral femur fractures with obvious deformity 

38
17. A
 30-year-old male is stabbed in the right chest. On arrival in the emergency b. immediate needle thoracentesis 

department, he is very short of breath. His heart rate is 120 and blood pressure is c. chest tube insertion 

80/50 mm Hg. His neck veins are flat. On auscultation of the chest, there is
diminished air entry on the right side, and there is dullness posteriorly on d. reassess the position of the endotracheal tube 

percussion, These findings are most consistent with: e. obtain a chest x-ray 


a. tension pneumothorax 

20. Which one of the following statements is true?
b. pericardial tamponade 

c. hypovolemia from liver injury 
 a. Elevated intracranial pressure will not affect cerebral perfusion. 


d. hemothorax 
 b. Cerebrospinal fluid cannot be displaced from the cranial vault. 


e. spinal cord injury 
 c. Cerebral blood flow is increased when the PacO2 is below 30 mm Hg. 

d. Autoregulation of cerebral blood flow normally occurs between mean arterial
18. A specific aspect of the treatment of thermal injuries is: pressures of 50 to 150 mm Hg. 

e. Hypotonic fluids should be used to limit brain edema in patients with severe
a. chemical burns require the immediate removal of clothing. 

head 
 injury. 

b. patients who sustain thermal injury are at lower risk for hypothermia.
c. patients with circumferential truncal burns need prompt fasciotomies.
21. A
 30-year-old male presents with a stab wound to the abdomen. Blood pressure
d. electrical burns are associated with extensive skin necrosis. 

is 85/60 mm Hg, heart rate is 130, respiratory is rate 25, and Glasgow Coma Scale
e. the Parkland formula should be used to determine adequacy of resuscitation.
score is 14. Neck veins are flat, and chest examination is clear with bilateral breath
Sounds. Optimal resuscitation should include:
19. A
 15-year-old male is brought to the emergency department after being involved
in a motor vehicle crash. He is unconscious and was intubated at the scene by a. transfusion of fresh frozen plasma and platelets. 

emergency medical personnel. Upon arrival at the emergency department, the
b. 500 mL of hypertonic saline and transfusion of packed red blood cells. 

patient's oxygen saturation is 92%, heart rate is 96, and blood pressure is 150/85
mm Hg. Breath sounds are decreased on the left side of the thorax. The next step c. resuscitation with crystalloid and packed red blood cells until base excess is
is: normal. 

a. immediate needle cricothyroidotomy 
 d. preparation for laparotomy while initiating fluid resuscitation. 


39
e. fluid resuscitation and angioembolization. 
 b. Cut-down at the ankle is the preferred initial access technique. 

c. Internal jugular cannulation is the next preferred option when percutaneous
22. Initial resuscitation in adult trauma patients should: venous access fails. 


a. be with 1­2 liters of crystalloid, monitoring the patient’s response. 
 d. Intraosseous cannulation should be the first choice for access. 


b. use crystalloid to normalize blood pressure. 
 e. Blood transfusion can be delivered through intraosseous access. 


c. use permissive hypotension in patients with head injury. 



25. A
 35-year-old female sustains multiple injuries in a motor vehicle crash and is
d. be with a non-blood colloid solution. 

transported to a small hospital in full spinal protection. She has a GCS score of 4
e. be a minimum of 2 liters of crystalloid in all trauma patients prior to and is being mechanically ventilated. Intravenous access is established and
administering blood. 
 warmed crystalloid is infused. She remains hemodynamically normal and full
spinal protection is maintained. Preparations are made to transfer her to another
23. A 25-year-old male is brought to the emergency department following a bar fight. facility for definitive neurosurgical care. Prior to transport, which of the following
tests or treatments is mandatory?
He has
 an altered level of consciousness, opens his eyes on command, moans
without forming discernible words, and localizes to painful stimuli. Which one of a. FAST exam 

the following statements concerning this patient is true?
b. Chest x-ray 

a. Mandatory intubation to protect his airway is required. 

c. lateral cervical spine x-ray 

b. His Glasgow Coma Scale suggests a severe head injury. 

c. His level of consciousness can be solely attributed to elevated blood alcohol. d. administration of methylprednisolone 

d. CT Scanning is an important part of neurological assessment. 
 e. computerized tomography of the abdomen 

e. A 2 L fluid bolus is indicated. 


26. A 23-year-old male is stabbed below the right nipple. He is alert, and his oxygen
24. Which one of the following statements is true regarding access in pediatric
saturation is 98%. Chest tube was placed for treatment of a hemopneumothorax.
resuscitation?
Blood pressure is 90/60 mm Hg after administration of 1 L of crystalloid solution.
a. Intraosseous access should only be considered after five percutaneous
What is the next step in treatment?
attempts. 

a. Place a left-sided chest tube. 

40
b. Re-examine the chest. 
 29. A
 70-year-old male suffers blunt chest trauma after being struck by a car. On
c. Insert central venous catheter. 
 presentation, his Glasgow Coma Scale score is 15, blood pressure is 145/90 mm
Hg, heart rate is 72, respiratory rate is 24, and oxygen saturation on 5 L is 91%.
d. Perform CT scan of the abdomen and pelvis. 
 Chest x-ray demonstrates multiple right-sided rib fractures. ECG demonstrates
e. Prepare for urgent thoracotomy. 
 normal sinus rhythm with no conduction abnormalities. Management should
include:

27. A
 22-year-old male is assaulted in a bar. A semi-rigid cervical collar is applied, a. placement of a 22-French, right-sided chest tube. 

and he is immobilized on a spine board. On initial examination, his vital signs are
b. serial troponins and cardiac monitoring. 

normal, and his Glasgow Coma Scale score is 15. Which of the following is an
indication for CT in this patient with possible minor traumatic brain injury? c. thoracic splinting, taping, and immobilization. 

d. monitored intravenous analgesia. 

a. presence of hemotympanum 

e. bronchoscopy to exclude tracheobronchial injury. 

b. blood alcohol concentration of 0.16% (160 mg/100 ml) 

c. presence of an isolated 10 cm scalp laceration 
 30. A 15-year-old male presents following a motorcycle crash. Initial examination
d. presence of a mandibular fracture 
 reveals normal vital signs. There is a large bruise over his epigastrium that extends
to the left flank. He has no other apparent injuries. A CT scan of the abdomen
e. history of assault 
 demonstrates a ruptured spleen surrounded by a large hematoma and fluid in the
pelvis. The next step in this patient's management is:
28. Supraglottic airway devices:
a. splenic artery embolization. 

a. are equivalent to endotracheal intubation. 

b. pneumococcal vaccine. 

b. require neck extension for proper placement. 

c. transfer to a pediatrician. 

c. are preferable to endotracheal intubation in a patient who cannot lie flat. 

d. urgent laparotomy. 

d. are of value as part of a difficult or failed intubation plan. 

e. surgical consultation. 

e. provide one form of definitive airway. 

31. A
 30-year-old female is brought to the emergency department after being

41
injured in a motor vehicle crash. Her initial blood pressure is 90/60 mm Hg, and d. control of hemorrhage. 

her heart rate is 122 beats per minute. She responds to the rapid infusion of 1 liter
of crystalloid Solution with a rise in her blood pressure to 118/88 mm Hg and a e. improve long-term function. 

decrease in her heart rate to 90 beats per minute. Her pressure then suddenly
decreases to 96/66 mm Hg. The least likely cause of her hemodynamic change 34. Which one of the following statements regarding genitourinary injuries is true?
is:
a. Urethral injuries are associated with pelvic fractures. 

a. traumatic brain injury. 
 b. All patients with microscopic hematuria require evaluation of the
b. ongoing blood loss. 
 genitourinary 
 tract. 

c. blunt cardiac injury. 
 c. Patients presenting with gross hematuria and shock will have a major renal
injury 
 as the source of hemorrhage. 

d. inadequate resuscitation. 

d. Intraperitoneal bladder injuries are usually managed definitively with a urinary
e. tension pneumothorax. 


 catheter. 

e. Urinary catheters should be placed in all patients with pelvic fractures during
32. Limb-threatening extremity injuries:
the 
 primary survey. 

a. require a tourniquet. 

b. are characterized by the presence of ischemic or crushed tissue. 
 35. A 21-year-old male athlete is involved in a motorcycle crash. When he arrives in
c. should be definitively managed by application of a traction splint. 
 the emergency department, he shouts that he cannot move his legs. On physical
examination, there are no abnormalities of the chest, abdomen, or pelvis. The
d. are rarely present without an open wound. 
 patient has no sensation in his legs and cannot move them, but his arms are
e. indicate a different order of priorities for the patient's initial assessment and moving. The patient's respiratory rate is 22, heart rate is 88, and blood pressure
resuscitation. 
 is 80/60 mm Hg. He is pale and sweaty. What is the most likely cause of his
condition?
33. The first priority in the management of a long bone fracture is:
a. neurogenic shock

a. reduction of pain. 

b. cardiac tamponade

b. prevention of infection in case of an open fracture. 

c. myocardial contusion
c. prevention of further soft tissue injury. 
 d. hyperthermia
e. abdominal hemorrhage
42
c. should be combined with clinical exam, AP and odontoid, or CT. 

36. A 27-year-old male presents following a motorcycle crash. He complains of the
inability to move or feel his legs. His blood pressure is 80/50 mm Hg, heart rate is d. are indicated in all trauma patients. 

70, respiratory rate is 18, and Glasgow Coma Scale score is 15. Oxygen e. require the following films: oblique views, AP, odontoid, and flexion and
Saturation is 99% on 21 nasal prongs. Chest x-ray, pelvic X-ray, and FAST are extension views prior to spinal clearance in trauma patients. 

normal. Extremities are normal. His management should be:
a. 2L of IV crystalloid and two units of pRBCs (packed red blood cells). 

39. A
 30-year-old male is brought to the hospital after falling 6 meters (20 feet).
b. 2 L of IV crystalloid, mannitol, and IV steroids. 
 Inspection reveals an obvious flail chest on the right. The patient is tachypneic.
c. 1 unit of albumin and compression stockings. 
 Breath sounds are present and symmetrical. There is no significant
hyperresonance or dullness. Arterial blood gases obtained while the patient
d. vasopressors and laparotomy. 
 receives oxygen by face mask are: Pao2 of 45 mm Hg (6 kPa), PaCO 2 of 28 mm
e. 2 L of crystalloid and vasopressors if BP does not respond. 
 Hg (3.7 kPa), and pH of 7.47. The component of injury that is most likely
responsible for the abnormalities in the patient’s blood gases is:
a. hypoventilation. 

b. hypovolemia. 

37. Which one of the following physical findings does not suggest spinal cord injury as
the cause of hypotension? c. small pneumothorax. 

a. priapism 
 d. pulmonary contusion. 

b. bradycardia 
 e. flail chest. 

c. distended neck veins 

40. An 82-year-old male falls down five stairs and presents to the emergency
d. diaphragmatic breathing 

department. All of the following are true statements regarding his condition
e. ability to flex forearms but inability to extend them 
 compared to a younger patient with similar mechanism, except?

38. Lateral cervical spine films: a. He is more likely to have had a contracted circulatory Volume prior to his
injury. 

a. must be performed in the primary survey. 
 b. His risk of cervical spine injury is increased due to degeneration, stenosis,

b. can exclude any significant spinal injury. 
 and 
 loss of disk compressibility. 


43
c. Intracranial hemorrhage will become symptomatic more quickly. 
 3. Bronchial intubation of the right or left mainstem bronchus can easily occur during
infant endotracheal intubation because
d. His risk of occult fractures is increased. 

e. His risk of bleeding may be increased. 
 a. the trachea is relatively short. 

b. the distance from the lips to the larynx is relatively short. 

ATLS Practice Test 4
c. the use of tubes without cuffs allows the tube to slip distally. 

1. A 25-year-old man, injured in a motor vehicular crash, is admitted to the emergency d. the mainstem bronchi are less angulated in their relation to the trachea. 

department. His pupils react sluggishly and his eyes open to painful stimuli. He
e. so little friction exists between the endotracheal tube and the wall of the
does not follow commands, but he does moan periodically. His right arm is
deformed and does not respond to painful stimulus; however, his left hand reaches trachea. 

toward it. Both legs are stiffly extended. His Glasgow Coma Scale score is
a. two. 
 4. Which one of the following statements regarding abdominal trauma in the pregnant
patient is TRUE?
b. four. 

c. six. 
 a. The fetus is in jeopardy only with major abdominal trauma. 

d. nine. 
 b. Leakage of amniotic fluid is an indication for hospital admission. 

e. twelve. c. Indications for peritoneal lavage are different from those in the nonpregnant
patient. 

2. Which one of the following statements concerning massive hemothorax is TRUE?
d. Penetration of an abdominal hollow viscus is more common in late than in
a. It is usually caused by blunt thoracic trauma. 

early pregnancy. 

b. It is commonly confused with a pneumothorax. 
 e. The secondary survey follows a different pattern from that of the nonpregnant
c. The diagnosis should be confirmed by upright, plain chest roentgenograms patient. 

prior to 
 treatment. 

d. The initial draining of 1,000 mL of blood after chest tube insertion requires
immediate 
 thoracotomy. 

e. The condition should be suspected in situations associated with shock and
unilateral absent 
 breath sounds. 
 5. The first maneuver to improve oxygenation after chest injury is
44
a. intubate the patient. e. cannot be explained by his observed injuries unless a major arterial injury
b. assess arterial blood gases. exists. 

c. administer supplemental oxygen. 

d. ascertain the need for a chest tube.
 8. Prior to passage of a urinary catheter, it is essential to

e. obtain a lateral cervical spine roentgenogram. 



a. examine the abdomen. 


6. A 17-year-old helmeted motorcyclist loses consciousness when he is struck broad b. determine pelvic stability. 

side by an automobile at an intersection. He arrives in the emergency department c. examine the rectum and perineum. 

with a blood pressure of 140/92, pulse rate of 88 beats per minute, a respiratory
rate of 18 breaths per minute, and a Glasgow Coma Scale score of seven. d. perform a retrograde urethrogram. 

Appropriate initial immobilization of this patient should include a semi-rigid cervical e. know the history and mechanism of injury. 

collar and

9. The best guide for adequate fluid resuscitation of the burn patient is
a. a scoop stretcher.
b. a long spine board.

a. adequate urinary output. 

c. a short spine board.
b. reversal of systemic acidosis. 

d. cervical traction tongs.

c. normalization of the heart rate. 

e. pneumatic antishock garment. 

d. a normal central venous pressure. 

7. A 34-year-old man is brought to the hospital after being pinned to the wall of a e. total fluids in 24 hours = 4 x weight (kg) x percent body surface area burned.
building by a cement truck. He is in obvious shock, and has deformities and marked
swelling of both thighs, although no open wounds are present. His shock 10. The LEAST likely cause of a depressed level of consciousness in the
multisystem injured patient is
a. cannot be explained without concomitant pelvic fracture. 

a. shock. 

b. signifies a loss of approximately 15% of his blood volume. 

b. head injury. 

c. is consistent with blood loss from bilateral femoral fractures. 

c. hyperglycemia. 

d. will likely be reversed if appropriate traction splints are applied. 

45
d. impaired oxygenation. 
 13. For the trauma patient with cerebral edema, hypercarbia should be avoided to
prevent
e. alcohol and other drugs. 

a. metabolic acidosis.
b. respiratory acidosis.

c. cerebral vasodilatation. 

d. neurogenic pulmonary edema.
11. Establishing a diagnosis of shock must include
 e. reciprocal high levels of PaCO2. 


a. confirming hypoxemia.
b. the finding of acidosis.

14. A 23-year-old man sustains four stab wounds to the right upper hemithorax during
c. confirming increased vascular resistance.
 an altercation and is brought by ambulance to a community hospital. The wounds
d. documenting hypotension and low cardiac output.
 are all above the nipple. He is endotracheally intubated, closed tube thoracostomy
is performed, and two liters of Ringer's lactate solution are infused through two
e. evidence of inadequate perfusion of the body's organs. 
 large-calibre IVs. His blood pressure now is 60/0, pulse rate is 160 beats per
minute, and respiratory rate is 14 breaths per minute (ventilated with 100% O 2).
12. A seven-year-old boy is brought to the emergency department by his parents The most appropriate next step in managing this patient is
several minutes after he fell through a window. He is bleeding profusely from a
6 cm (2.4 inch) wound of his medial right thigh. Immediate management of the a. angiography. 

wound should consist of
b. thoracotomy. 

a. application of a tourniquet. 
 c. CT of the chest. 

b. direct pressure on the wound. 
 d. application of PASG. 

c. apply a hemostat to bleeding vessels. 
 e. immediate transfer to another facility. 

d. direct pressure on the femoral artery at the groin. 

15. An 80-year-old female presents after being struck by a scooter. Her respiratory
e. application of the pediatric PASG and inflation of the right leg
rate is 20, heart rate is 86, and blood pressure is 100/70 mm Hg. Her Glasgow
compartment.
 Coma Scale score is 12, and she is bleeding from a scalp laceration. Which one
46
of the following treatment steps is the priority? 18. A patient is brought to the emergency department 20 minutes after a motor
vehicular crash. He is conscious and there is no obvious external trauma. He
a. administering 1 L of crystalloid 
 arrives at the hospital intubated and completely immobilized on a long spine board.
His blood pressure is 60/40 and his pulse rate is 70 beats per minute. His skin is
b. controlling bleeding from the scalp 
 warm and he has no rectal tone. Which one of the following statements is TRUE?
c. performing a neurological examination 

a. Vasoactive medications have no role in early management.
d. obtaining a CT scan of the head 

b. The hypotension should be managed with volume resuscitation alone.

e. obtaining detailed information about medication and previous history 

c. Flexion and extension views of the c-spine should be performed early.

d. Occult abdominal visceral injuries can be excluded as a cause of
16. The cause of upper airway obstruction most likely to be alleviated by a properly
placed oral airway is hypotension.

e. Flaccidity of the lower extremities and loss of deep tendon reflexes are
a. foreign body. 
 expected. 


b. facial fractures. 

c. copious, tenacious oral secretions. 

d. hematoma of the posterior pharynx. 

e. posterior displacement of tongue. 
 19. A 25-year-old female in the third trimester of pregnancy is involved in a motor
vehicle crash in which she is the front seat passenger. The driver is dead on scene.
17. The primary indication for transferring a patient to a higher level trauma center is Which one of the following statements concerning this patient is true?

a. multisystem injuries, including severe head injury.
 a. The patient should be told as soon as possible that her husband is dead. 


b. unavailability of a surgeon or operating room staff.
 b. Palpation of clearly definable fetal parts allows for assessment of the stage
of pregnancy. 

c. unavailability of an intensive care unit bed or ICU staff. 

c. The thick-walled uterus provides protection for the fetus. 

d. resource limitations as determined by the referring physician.

e. a widened mediastinum on chest roentgenogram after blunt thoracic trauma. d. Absence of fetal movements indicates maternal shock. 


47
e. Clear fluid from the vagina is an indication for hospital admission. 
 b. confusion. 

c. hypotension. 

20. Cervical spine injury
d. tachycardia. 


a. is excluded by a normal neurologic examination. 
 e. blood transfusion requirement. 



b. is not present if the patient has normal range of neck motion. 

23. Which one of the following statements concerning intraosseous infusion in children
c. can be detected safely by careful flexion and extension of the neck. 
 is TRUE?
d. can be excluded by a crosstable lateral roentgenogram of the c-spine. 

a. Only crystalloid solutions may be safely infused through the needle.

e. may be first manifested by neurologic deficit after movement of the neck. 

b. Aspiration of bone marrow confirms appropriate positioning of the needle.

21. Typical pelvic fracture patterns and injuries following major mechanisms of injury c. Intraosseous infusion is the preferred route for volume resuscitation in small
include: children. 

d. Intraosseous infusion may be utilized indefinitely in the management of
a. lateral compression forces, resulting in open book pelvic fractures. 
 injured children.
b. anterior-posterior compression, resulting in shifting of the iliac bones e. Swelling in the soft tissue around the intraosseous site is not a reason to
vertically. 
 discontinue 
 infusion. 

c. Vertical displacement of the hemipelvis, typically resulting in disruption of the

 sacrospinous ligaments. 
 24. A 26-year-old seat-belted driver is brought to the hospital after a car crash. Primary
survey reveals no evidence of serious injury except for diffuse, mild abdominal
d. direct blows to the iliac crest of the pelvis, resulting in sacral fractures. 
 tenderness. Bowel sounds are hypoactive and liver dullness is questionable.
e. posterior displacement of the knee and femur in motor vehicle crashes, Abdominal films reveal free air. The patient should
resulting in 
 acetabular fractures with anterior hip dislocation. 

a. undergo peritoneal lavage. 

22. Hemorrhage of 20% of the patient's blood volume is associated usually with b. undergo prompt laparotomy. 

c. be carefully observed for further evidence of intra-abdominal injury. 

a. oliguria. 

d. have a contrast roentgenographic study of the gastrointestinal tract. 

48
e. be suspected of having a ruptured diaphragm and accompanying 27. Which of the following injuries is most likely to be missed by FAST or DPL?
pneumothorax. 

a. Mesenteric laceration 

25. A 31-year-old woman sustains multiple injuries in a motor vehicle collision. She is b. Splenic capsular laceration 

hemodynamically normal, not in respiratory distress, GCS is 8, and has facial
fractures. Breath sounds are slightly decreased in the right hemithorax. Bowel c. Ileal rupture 

sounds are normal. She has a right open femur fracture. Chest film shows a small d. Hepatic fractures 

right pneumothorax and a widened mediastinum. Surgical specialty capabilities
are not available locally, but are available 50 miles away. Prior to transport, all of e. Duodenal rupture 

the following should be performed EXCEPT

28. In managing the head-injured patient, the most important initial step is to :

a. chest tube insertion.
 a. secure the airway. 



b. endotracheal tube insertion.
 b. immobilize the neck. 

c. CT scan of the head.
 c. support the circulation. 

d. speak directly to the receiving physician.
 d. control scalp hemorrhage. 

e. infusion of crystalloid. 
 e. determine the Glasgow Coma Scale score. 


26. A four-year-old girl, weighing approximately 20 kg (44 pounds), is admitted in


29. A
 previously healthy, 70 kg (175 pound) man suffers an estimated acute blood
shock after an automobile crash. How much crystalloid should be given as an initial
fluid bolus? loss of two liters. Which one of the following statements apply to this patient?

a. His pulse pressure will be widened. 



a. 200 mL. 

b. His urinary output will be at the lower limits of normal. 

b. 400 mL. 

c. He will have tachycardia, but no change in his systolic blood pressure. 

c. 440 mL. 

d. His systolic blood pressure will be decreased with a narrowed pulse pressure.
d. 600 mL. 
 e. His systolic blood pressure will be maintained with an elevated diastolic
e. 880 mL. 
 pressure. 

49
e. the patient complains of potential pressure sores due to the spine board. 

30. The physiologic hypervolemia of pregnancy has clinical significance in the
management of the severely injured, gravid woman by
33. The most important, immediate step in the management of an open
a. reducing the need for blood transfusion. 
 pneumothorax is
b. increasing the risk of pulmonary edema. 

a. endotracheal intubation. 

c. complicating the management of closed head injury. 

b. operation to close the wound. 

d. increasing the volume of blood loss to produce shock. 

c. placing a chest tube through the chest wound. 

e. reducing the volume of crystalloid required for resuscitation. 

d. placement of an occlusive dressing that is open on one side over the wound.
31. Which one of the following findings should prompt immediate management e. initiation of two, large-calibre IVs with Ringer's lactate. 

during the primary survey?
a. Distended abdomen 

b. Glasgow Coma Scale Score of 11 

c. Pulse rate of 120 beats per minute 

34. Important screening roentgenograms to obtain in the multiple-system trauma
o o
d. Temperature of 36.5 C (97.8 F) 
 patient are
e. Respiratory rate of 32 breaths per minute. 

a. skull, chest, and abdomen. 

32. During the primary and secondary surveys, the injured patient should be b. chest, abdomen, and pelvis. 

completely immobilized until
c. cervical spine, chest, and pelvis. 


a. the neurologic examination has been completed. 
 d. skull, cervical spine, and chest. 


b. the patient is transferred to a definitive care area. 
 e. cervical spine, chest, and abdomen. 


c. the patient is able to indicate that he has no neck pain. 



35. All of the following statements regarding pulse oximetry are true EXCEPT:

d. a spinal fracture has been excluded clinically or by roentgenograms. 


50
a. Excessive surrounding room light can interfere with the accuracy of the a. undergo exploratory celiotomy (laparotomy).

readings.
b. Significant levels of dysfunctional hemoglobin can affect the accuracy of the b. be treated for neurogenic shock.

readings. c. be treated for hypovolemic shock.

c. It provides a continuous, noninvasive measurement of the partial pressure of
oxygen. d. undergo immediate nasotracheal intubation.

d. It is dependent on differential light absorption by oxygenated and e. be placed in cervical traction tongs before any other treatment is instituted.
deoxygenated 
 hemoglobin.
 

e. It provides a continuous, noninvasive measurement of pulse rate that is
updated with each 
 heart beat. 
 38. A 52-year-old woman sustains 50% total body-surface flame burns in an
explosion. She has circumferential burns around the chest and of both upper arms.
Adequate resuscitation is initiated. She is nasotracheally intubated and is being
36. A 56-year-old man is thrown violently against the steering wheel of his truck during
mechanically ventilated. Her carboxyhemoglobin level is 10%. Her arterial blood
a motor vehicular crash. On arrival in the emergency department he is diaphoretic
gas analysis reveals PaO2 of 40 mm Hg, PaCO2 of 60 mm Hg, and pH of 7.25.
and complains of chest pain. His blood pressure is 60/40 and his respiratory rate
Appropriate immediate management at this time is to
is 40 breaths per minute. Which of the following would best differentiate cardiac
tamponade from tension pneumothorax as the cause of his hypotension?
a. ensure adequate tissue perfusion. 

a. Tachycardia 
 b. increase the rate of fluid resuscitation. 

b. Pulse volume 
 c. add positive end-expiratory pressure (PEEP). 

c. Breath sounds 
 d. reassess for the presence of a pneumothorax. 

d. Pulse pressure 
 e. administer intravenous narcotics in small amounts. 

e. Jugular venous pressure 


37. An 18-year-old man is brought to the hospital after smashing his motorcycle into
a tree. He is conscious and alert, but paralyzed in both arms and legs. His skin is
pale and cold. He complains of thirst and difficulty in breathing. His airway is clear.
His blood pressure is 60/40 and his pulse rate is 140 beats per minute. Breath 39. A 25-year-old man is brought to the hospital after being involved in a motor
sounds are full and equal bilaterally. He should vehicular crash when his car struck a bridge abutment. He is intoxicated, has a
Glasgow Coma Scale score of 13, and complains of abdominal pain. His blood
51
pressure was 80 mm Hg systolic by palpation on admission to the hospital, but it 1. Thoracic trauma. Chest tube insertion.
rapidly increased to 110/70 with the administration of intravenous fluids. His heart
rate is now 120 beats per minute. The chest roentgenogram shows loss of the A 22 year old man is hypotensive and tachycardic after a shotgun wound to the left
aortic knob, widening of the mediastinum, no rib fractures, and no shoulder. His blood after initial IV fluid resuscitation, a closed tube thoracostomy is
hemopneumothorax. Contrast CT angiography performed for decreased left breath sounds

a. reexamine the chest



a. is not indicated.

b. perform an aortogram

b. should be performed after tube thoracostomy.

c. is not necessary if non-contrast CT scan of the chest is normal. c. obtain a CT scan of the chest

d. should be performed after a FAST scan.
 d. obtain arterial blood gas analyses

e. is positive for aortic rupture in 80% of similar cases. 
 e. perform tranesohageal echocardiography

answer: a.
 info: chest tube insertion, p. 108.



40. A 32-year-old man is trapped from the waist down beneath his overturned car
for a period exceeding eight hours. On arrival in the emergency department, both
2. Musculoskeletal trauma. Extremity trauma.
lower extremities are cool, mottled, insensate, and motionless. Despite normal
vital signs, pulses cannot be palpated below the femoral vessels and the muscles A construction worker falls two stories from a building and sustains bilateral calcaneal
of the lower extremities are firm and hard. During the initial management of this fractures. In the emergency department, he is alert, vital signs are normal, and he is
patient, which of the following is most likely to improve the chances for limb complaining of severe pain in both heels and his lower back. Lower extremity pulses
salvage? are strong and there is no other deformity. The suspected diagnosis is most likely to
be confirmed by
a. Applying skeletal traction 

a. angiography

b. Administering anticoagulant drugs 

b. compartment pressures

c. Administering thrombolytic therapy 

c. retrograde urethrogram

d. Performing lower extremity fasciotomies 

d. Doppler-ultrasound studies
e. Immediately transferring the patient to a trauma center 
 e. complete spine x-ray series

ATLS: Pretests answer: e.


52
a. his pulse pressure will be widened

3. Trauma in women. b. his urinary output will be at the lower limits of normal


During the third trimester of pregnancy, all of the following changes occur normally c. he will have tachycardia, but no change in his systolic blood pressure

EXCEPT a d. his systolic blood pressure will be decreased with a narrowed pulse ressure
(true)
a. decrease in PaCO2 e. his systolic blood pressure will be maintained with an elevated diastolic
b. decrease in leukocyte count
 pressure

c. reduced gastric emptying rate
 answer: d. info: p. 61.


d. diminished residual lung volume
e. diminished elvic ligament tension 6. Trauma in Women.

answer: b. info: p. 261. The physiologic hypervolemia of pregnancy has clinical significance in the
management of the severely injured, gravid woman by
4. Head Trauma.
 In managing the head injured patient, the most important initial
a. reducing the need for blood transfusion

step is to
b. increasing the risk of pulmonary edema

a. secure the airway

c. complicating the management of closed head injury

b. obtain c-spine film
 d. reducing the volume of crystalloid required for resuscitation
c. support circulation
 e. increasing the volume of blood loss to
 produce maternal hypotension
d. control scalp hemorrhage
e. determine the GCS score answer: e.
 info: p. 261.


answer: a. info: p. 154.


7. Thermal Injuries. Injury Due to Burn and Cold.

5. Shock.
The best guide for adequate fluid resuscitation of the burn patient is
A previously healthy, 70kg (154 pound) man suffers an estimated acute blood loss of
2 liters. Which one of the following statements applies to this patient? a. adequate urinary output


53
b. reversal of systemic acidosis
 a. application of a tourniquet

c. normalization of the heart rate
 b. direct pressure on the wound

d. a normal central venous pressure
 c. packing the wound with gauze

e. 4mL/kg/percent body burn/24 hours d. direct pressure on the femoral artery at the groin
e. debridement of devitalized tissue
answer: a.
 info: p. 216-217.

answer: b info: p. 79.

8. Shock.

10. Head injury.
Establishing a diagnosis of shock must include
For the patient with severe traumatic brain injury, profound hypocarbia should be
a. hypoxemia
 avoided to prevent

b. acidosis
 a. respiratory alkalosis



c. hypotension
 b. metabolic acidosis

d. increased vascular resistance
 c. cerebral vasoconstriction with diminished perfusion
e. evidence of inadequate organ perfusion d. neurogenic pulmonary edema
e. shift of the oxyhemoglobin dissociation curve
answer: e. info: p. 58.
answer: c
 info: p. 136, 137.
9. Musculoskeletal trauma. Extremity Trauma.
Carbon dioxide is perhaps the most potent available modulator of cerebrovascular
A 7 year old boy is brought to the emergency department by his parents several
tone and thus cerebral blood flow (CBF). Hypercarbia and hypoxia are both potent
minutes after he fell through a window. He is bleeding profusely from a 6-cm wound
cerebral vasodilators that result in increased cerebral blood flow and volume and,
of his medial right thigh. Immediate management of the wound should consist of
potentially, increased ICP; thus, they must be avoided. Orotracheal intubation allows
54
for airway protection in patients who are severely obtunded and allows for better e. the secondary survey follows a different pattern from that of the nonpregnant
control of oxygenation and ventilation. patient

11. Abdominal trauma. answer: b.
 info: p. 265.



A 25 year old man is brought to a hospital with a general surgeon after being involved
in a motor vehicle crash. He has a GCS of 13 and complains of abdominal pain. His 13. Thoracic trauma.

blood pressure was 80 mm Hg systolic by palpation on arrival at the hospital, but
increases to 110/70 mm Hg with the administration of 2 liters of intravenous fluid. His The first maneuver to improve oxygenation after chest injury is
heart rate remains 120 beats per minute. Computed tomography shows an aortic
injury and splenic laceration with free abdominal fluid. His blood pressure falls to 70 a. intubate the patient

mm Hg after CT. The next step is b. assess arterial blood gases

c. administer supplemental oxygen
a. contrast angiography

d. ascertain the need for a chest tube
b. transfer to higher level trauma center e. obtain a chest x-ray
c. exploratory laparotomy

answer: c.
d. transfuse packed red blood cells

e. transesophageal echocardiography

answer: c. info: p. 12. 14. Head trauma.

12. Which one of the following statements regarding abdominal trauma in the A 25 year old man, injured in a motor vehicular crash, is admitted to the emergency
pregnant patient is true? department. His pupils react sluggishly and his eyes open to painful stimuli. He does
not follow commands, but he does moan periodically. His right arm is deformed and
a. the fetus is in jeopardy only with major abdominal trauma
 does not respond to painful stimulus; however, his left hand reaches purposefully
toward the painful stimulus. Both legs are stiffly extended. His GCS Score is
b. leakage of amniotic fluid is an indication for hospital admission

c. indications for peritoneal lavage are different from those in the nonpregnant a. 2
b. 4
patient

c. 6
d. penetration of an abdominal hollow viscus is more common in late than in d. 9
early pregnancy e. 12
55
answer: d.
 info: p. 138.
 answer: e.
 info: p. 79.


15. Trauma in Women. 17. Thoracic trauma.



A 20 year old woman, at 32 weeks gestation, is stabbed in the upper right chest. In
The most important, immediate step in the management of an open pneumothorax is
the emergency department, her blood pressure is 80/60 mm Hg. She is gasping for
breath, extremely anxious, and yelling for help. Breath sounds are diminished in the
a. endotracheal intubation

right chest. The most appropriate first step is to
b. operation to close the wound

a. perform tracheal intubation

c. placing a chest tube through the chest wound

b. insert an oropharyngeal airway

d. placement of an occlusive dressing over the wound

c. perform needle decompression of the right chest
 e. initiation of 2, large-caliber IVs with crystalloid solution
d. manually displace the gravid uterus to the left side of the abdomen
e. initiate 2, large-caliber peripheral IV lines and crystalloid infusion answer: d. info: p. 87.

answer: c.
 info: p. 87.
 18. Tetanus immunization.


The following are contraindications for tetanus toxoid administration

16. Initial assessment and management. a. history of neurological reaction or severe hypersensitivity to the product
b. local side effects

Which one of the following findings in an adult should prompt immediate management
during the primary survey? c. muscular spasms

d. pregnancy
a. distended abdomen
 e. all of the above
b. glasgow coma scale score of 11

answer: a.
 info: p. 297.

c. temperature of 36.5C (97.8F)

19. Thoracic trauma.
d. heart rate of 120 beats per minute

e. respiratory rate of 40 breaths per minute A 56 year old man is thrown violently against the steering wheel of his truck during a

56
motor vehicle crash. On arrival in the emergency department he is diaphoretic and His wounds are all above the nipple. He is endotracheally intubated, closed tube
complaining of chest pain. His blood pressure is 60/40 mm Hg and his respiratory rate thoracostomy is performed, and 2 liters of crystalloid solution are infused through 2
is 40 breaths per minute.Which of the following best differentiates cardiac tamponade large-caliber IVs. His blood pressure now is 60/0 mmHg, heart rate is 160 beats per
from tension pneumothorax as the cause of his hypotension? minute, and respiratory rate is 14 breaths per minute (ventilated with 100% O2).
1500cc of blood has drained from the right chest. The most appropriate next step in
a. tachycardia managing this patient is to
b. pulse volume
c. breath sounds a. perform FAST

d. pulse pressure

b. obtain a CT of the chest

e. jugular venous pressure
c. perform an angiography

answer: c. info: p. 87. d. urgently transfer the patient to the operating room
e. immediately transfer the patient to a trauma center
20. Pediatric trauma. Trauma in extremes of age.

Bronchial intubation of the right or left mainstem bronchus can easily occur during answer: d.
 info: p. 90-91.

infant endotracheal intubation because
22. Airway and ventilatory management.
a. the trachea is relatively short

A 39 year old man is admitted to the emergency department after an automobile
b. the distance from the lips to the larynx is relatively short
 collision. He is cyanotic, has insufficient respiratory effort, and has a GCS score of 6.
His full beard makes it difficult to fit the oxygen facemask to his face. The most
c. the use of tubes without cuffs allows the tube to slip distally

appropriate next step is to
d. the mainstem bronchi are less angulated in their relation to the trachea
e. so little friction exists between the endotracheal tube and the wall of the a. perform a surgical cricothyroidotomy

trachea
b. attempt nasotracheal intubation

answer: a.
 info: p. 228.
 c. ventilate him with a bag-mask device until c-spine injury can be excluded

d. attempt orotracheal intubation using 2 people and inline stabilization of the
21. Thoracic trauma. cervical spine.
e. ventilate the patient with a bag-mask device until his beard can be shaved for
A 23 year old man sustains 4 stab wounds to the upper right chest during an better mask fit.
altercation and is brought by ambulance to a hospital that has full surgical capabilities.

57
answer: d.
 info: p. 33.
 e. topical application of silver sulfadiazine

answer: a info: p. 220.


23. Spine and Spinal Cord Trauma.
25. Musculoskeletal trauma. Extremity trauma.
A patient is brought to the emergency department 20 minutes after a motor vehicle
crash. He is conscious and there is no obvious external trauma. He arrives at the A 32 year old man's right leg is trapped beneath his overturned car for nearly 2 hours
hospital completely immobilized on a long spine board. His blood pressure is 60/40 before he is extricated. On arrival in the emergency departmnet, his right lower
mmHg and his heart rate is 70 beats per minute. His skin is warm. Which one of the extremity is cool, mottled, insensate, and motionless. Despite normal vital signs,
following statements is true? pulses cannot be palpated below the femoral vessel and the muscles of the lower
extremity are firm and hard. During the initial management of this patient, which of
a. vasoactive medications have no role in the patient's management
 the following is most likely to improve the chances for limb salvage?
b. the hypotension should be managed with volume resuscitation alone

a. applying skeletal traction

c. flexion and extension views of the c-spine should be performed early

b. administering anticoagulant drugs

d. occult abdominal visceral injuries can be excluded as a cause of
hypotension
 c. administering thrombolytic therapy

e. flaccidity of the lower extremities and loss of deep tendon reflexes are d. perform right lower extremity fasciotomy

expected e. immediately transferring the patient to a trauma center

answer: c.
 info: p. 167; 161.
 answer: d.
 info: p. 196-197.

24. Thermal injuries.
 26. Head trauma.

Which one of the following is the recommended method for initialy treating frostbite? A patient arrives in the emergency department after being beaten about the head and
face with a wooden club. He is comatose and has a palpable depressed skull fracture.
a. moist heat
 His face is swollen and ecchymotic. He has gurgling respirations and vomitus on his
face and clothing. The most appropriate step after clothing. The most appropriate step
b. early amputation
 after providing supplemental oxygen and elevating his jaw is to
c. padding and elevation

a. requires a CT scan

d. vasodilators and heparin


58
b. insert a gastric tube
 pressure is 100/74 mmHg, heart rate is 110 beats per minute, and respiratory rate is
18 breaths per minute. He has gross hematuria and a pelvic fracture. You decide to
c. suction the oropharynx
 transfer this patient to a facility capable of providing a higher level of care. The facility
d. obtain a lateral cervical spine x-ray is 128 km (80 miles) away. Before transfer, you should first
e. ventilate the patient with a bag-mask
a. intubate the patient

answer: c
b. perform diagnostic peritoneal lavage

27. Thoracic trauma. Transfer to Definitive Care.
c. apply the pneumatic antishock garment

A 22 year old man sustains a gunshot wound to the left chest and is transported to a d. call the receiving hospital and speak to the surgeon on call
small community hospital at which surgical capabilities are not available. In the e. discuss the advisability of transfer with the patient's family
emergency department, a chest tube is inserted and 700mL of blood is evacuated.
The trauma center accepts the patient in transfer. Just before the patient is placed in answer: c. info: p. 123.
the ambulance for transfer, his blood pressure decreases to 80/68 mmHg and his
heart rate increases to 136 beats per minute. The next step should be to 29. Shock.


a. clamp the chest tube
 Hemorrhage of 20% of the patient's blood volume is associated usually with

b. cancel the patient's transfer
 a. oliguria



c. perform an emergency department thoracotomy
 b. confusion

d. repeat the primary survey and proceed with transfer
 c. hypotension

e. delay the transfer until the referring doctor can contact a thoracic surgeon
d. tachycardia

answer: c. info: p. 270 - 274. e. blood transfusion requirement

28. Head trauma. answer: d.
 info: p. 61.



A 64 year old man, involved in a high-speed car crash, is resuscitated initially in a
small hospital with limited resources. He has a closed head injury with a GCS score 30. Intraosseous fluid resuscitation.

of 13. He has a widened mediastinum on chest x-ray with fractures of left ribs 2
through 4, but no pneumothorax. After infusing 2 liters of crystalloid solution, his blood Which one of the following statements concerning intraosseous infusion is true?

59
a. only crystalloid solutions may be safely infused through the needle
 answer: a.
 info: p. 142, 143, 145.
b. aspiration of bone marrow confirms appropriate positioning of the needle

32. Thoracic trauma.
c. intraosseous infusion is the preferred route for volume resuscitation in small
children
 A 33 year old woman is involved in a head-on motor vehicle crash. It took 30 minutes
to extricate her from the car. Upon arrival in the emergency department, her heart rate
d. intraosseous infusion may be utilized indefinitely
 is 120 beats per minute, BP is 90/70 mmHg, respiratory rate is 16 breaths per minute,
e. swelling in the soft tissues around the intraosseous site is not a reason to and her GCS score is 15. Examination reveals bilaterally equal breath sounds,
discontinue infusion anterior chest wall ecchymosis, and distended neck veins. Her abdomen is flat, soft,
and not tender. Her pelvis is stable. Palpable distal pulses are found in all 4
answer: b. the info: p.236. extremities. Of the following, the most likely diagnosis is

a. hemorrhagic shock
b. cardiac tamponade

c. massive hemothorax
31. Head injury. d. tension pneumothorax
e. diaphragmatic rupture
A young woman sustains a severe head injury as the result of a motor vehicle crash.
In the emergency department, her GCS is 6. Her blood pressure is 140/90 mmHg and
answer: b.
 info: p. 91-92.

her heart rate is 80 beats per minute. She is intubated and mechanically ventilated.
Her pupils are 3mm in size and equally reactive to light. There is no other apparent
33. Abdominal trauma.
injury. The most important principle to follow in the early management of her head
injury is to A hemodynamically normal 10 year old girl is admitted to the Pediatric Intensive Care
Unit (PICU) for observation after a Grade III (moderately severe) splenic injury has
a. avoid hypotension
 been confirmed by computed tomography (CT). Which of the following mandates
b. administer an osmotic diuretic
 prompt celiotomy (laparotomy)?

c. aggressively treat systemic hypertension
 a. A serum amylase of 200

d. reduce metabolic requirements of the brain
 b. A leukocyte count of 14,000



e. distinguish between intracranial hematoma and cerebral edema c. extraperitoneal bladder rupture


60
d. free intraperitoneal air demonstrated on follow-up CT
 c. insert bilateral thoracostomy tubes

e. a fall in the hemoglobin level from 12 g/dL to 8 g/dL over 24 hours d. maintain 100% oxygen and obtain immediate c- spine x-rays
e. inline immobilization and establish a definitive airway
answer: e.
 info: p. 121.

answer: e.
 info: p. 27-28.

34. Spine and spinal cord trauma.
36. Thermal injury. Injury due to burn and cold.
A 40 year old woman restrained driver is transported to the emergency department in
full spinal immobilization. She is hemodynamically normal and found to be paraplegic When applying the Rule of Nines to infants,
at the level of T10. Neurologic examination also determines that there is loss of pain
and temperature sensation with preservation of proprioception and vibration. These a. it is not reliable

findings are consistent with the diagnosis of b. the body is proportionally larger in infants than in adults
c. the head is proportionally larger in infants than in adults
a. central cord syndrome

d. the legs are proportionally larger in infants

b. spinal shock syndrome
 e. the arms are proportionally larger in infants
c. anterior cord syndrome

answer: c.
 info: p. 223.

d. complete cord syndrome
e. Brown-Sequard syndrome
37. Abdominal trauma.
answer: c.
 info: p. 163.
 A 60 year old man sustains a stab wound to the right posterior flank. Witnesses state
the weapon was a small knife. His heart rate is 90 beats per minute, blood pressure
35. Spine and spinal cord injury. is 128/72 mmHg, and respiratory rate is 24 breaths per minute. The most appropriate
action to take at this time is to perform
A trauma patient presents to your emergency department with inspiratory stridor and
a suspected c-spine injury. Oxygen saturation is 88% on high-flow oxygen via a
a. colonoscopy

nonrebreathing mask. The most appropriate next step is to:
b. barium enema

a. apply cervical traction

c. an intravenous pyelogram

b. perform immediate tracheostomy

d. serial physical examination


61
e. suture repair the wound and outpatient follow up e. chest CT

answer: d.
 answer: d. info: p. 90-91.

40. Pediatric trauma.


38. Trauma in Women.
A 6 year old boy walking across the street is struck by the front bumper of a sports
Which of the following situations requires Rh immunoglobulin administration to an
utility vehicle traveling at 32 kph (20 mph). Which one of the following statements is
injured woman?
true?

a. negative pregnancy test, Rh negative, and torso trauma



a. a flail chest is probable

b. positive pregnancy test, Rh positive, and has torso trauma

b. symptomatic cardiac contusion is expected

c. positive pregnancy test, Rh negative, and has torso trauma

c. pulmonary contusion may be present in the absence of rib fractures

d. positive pregnancy test, Rh positive, and has an isolated wrist fracture
e. positive pregnancy test, Rh negative, and has an isolated wrist fracture d. transection of the thoracic aorta more likely than in an adult patient

e. rib fractures are commonly found in children with this mechanism of injury
answer is: c.
 info: p. 265.

answer: c. info: p. 237.
39. Thoracic trauma.
ATLS 2013
A 22 year old female athlete stabbed in her left chest at the third interspace in the
anterior axillary line. On admission to the emergency department and 15 minutes after 1. A 20 year old athlete is involved in motorcycle crash. When he arrives in the ER.
the incident, she is awake and alert. Her heart rate is 100 beats per minute, blood He shouts that he cannot move his legs. On physical exam, there are no
pressure 80/60 mmHg, and respiratory rate 20 breaths per minute. A x-ray reveals a abnormalities of the chest, abdomen, or pelvis. The patient has no sensation in his
large left hemothorax. A left chest tube is placed with an immediate return 1600 mL legs and cannot move them, but his arms are moving. The patient's respiratory
of blood. The next management step for this patient is perform: rate is 23, heart rate 88, and blood pressure is 80/60 mm Hg. He is pale and
sweaty. What is most likely cause of his condition?
a. thoracoscopy
 a. Neurogenic shock.
b. Cardiac temponade.
b. an arch aortogram
 c. Myocardial contusion.
d. Hypothermia.
c. insert a second left chest tube

e. Abdominal hemorrhage.
d. prepare for an exploratory thoracotomy
62
2. A 22 year old male presents following a motorcycle crash. He complains of the 5. 28 year old male is brought to ER. He was involved in fight, in which he was
inability to move or feel his legs. His blood pressure is 80/50 mm Hg, heart rate is beaten with a wooden stick. His chest shows multiple severe bruises. Airway is
70, respiratory rate is 18. GCS is 15. Oxygen is 99%on 2L nasal prongs. Chest X- clear, respiratory rate is 22, heart rate is 126, and systolic blood pressure is 90
ray, pelvic X-ray, FAST are normal. Extremities are normal. His management mm Hg. Which of the following should be performed during the primary survey?
should be: a. GCS.
a. 2L of IV crystalloid and two units of pRBSs. b. Tetanus toxoid administration.
b. 2L of IV crystalloid, manitol and IV steroids. c. Cervical spine X-ray.
c. 1 unit of albumin and compression stockings. d. Blood alcohol level.
d. Vassopressors and laprotomy. e. Rectal exam.
e. 2L of crystalloid and vassopressors if BPdoes not respond.
6. Which one of the following injuries is addressed in the secondary survey?
3. Which of the following signs is least reliable for diagnosing esophageal intubation? a. Forearm fracture.
a. Symmetrical chest wall movement. b. Mid-thigh amputation.
b. End-tidal CO2. c. Open fracture with bleeding.
c. Bilateral breath sounds. d. Unstable pelvic fracture.
d. Oxygen saturation. e. Bilateral femur fractures with obvious deformity.
e. Chest X-ray demonstrating the ETT tip positioned above the carnia.
7. Which one of the following statements is true regarding access in pediatric
4. A 6 months old infant, being held in her mother's arms, is ejected on impact from resuscitation?
a vehicle that is struck head-on by an oncoming car travelling at 64 kph. The infant a. Intraosseous access should only be considered after five percutaneous
arrives in the ER with multiple facial injuries, is lethargic, and in severe respiratory attempts.
distress. Respiratory support is not effective using a bag mask device. And her b. Cut-down at the ankle is the preferred initial access technique.
oxygen saturation is falling. Repeated attempts at orotracheal intubation are c. Internal jugular cannulation is the next preferred option when percutaneous
unsuccessful. The most appropriate procedure to perform next is: venous access fails.
a. Administer hellox and epinephrine. d. Intraosseouscannulation should be the first choice for access.
b. Perform nasotracheal intubation. e. Blood transfusion can be delivered through intraosseous access.
c. Perform surgical cricothyriodotomy.
d. Repeat orotracheal intubation. 8. A 35 year old female sustains multiple injuries in a motor vehicle crash and is
e. Perform needle cricothyroidtomy with jet insufflations. transported to a small hospital in full spinal protection. She has a GCS of 4 and is
being mechanically ventilated. Intravenous access is established and warmed
crystalloid is infused. She remains hemodynamically normal and full spinal
protection in maintained. Preparations are made to transfer her to another facility
63
for definitive neurosurgical care. Prior to transport, which of the following tests or a. Pelvic injury can be ruled out based on the mechanism of injury.
treatments is mandatory? b. Blood loss from the lower limbs is the most likely cause of hypotension.
a. FAST exam. c. X-ray of the chest and pelvis are important adjacent in his initial
b. Chest X-ray. assessment.
c. Lateral cervical spine X-ray. d. Spinal cord injury is most likely cause of his hypotension.
d. Administration of methiprednisolone. e. Aortic injury is the most likely cause of his tachycardia.
e. CT of abdomen.
12. A 25 year old female in the third trimester of pregnancy is brought to ER following
9. A 23 year old male is stabbed below the right nipple. He is alert, and his oxygen a high-speed motor vehicle crash. She is conscious and immobilized on long spine
is 98%. Chest tube was placed for treatment of hemopnueunthorax. BP 90/60 mm board. RR is 24, HR is 120, and BP is 70/50. Labs show a PaCO2 of 40 mm Hg.
Hg after 1L of crystalloid solution. What is the next step in treatment? Which one of the following statements concerning this patient is true?
a. Place a left-sided chest tube. a. Fetal assessment should take priority.
b. Re-examine the chest. b. Log rolling the patient to the right will decompress the vena cava.
c. Inscert central venous catheter. c. Rh-immunoglubulin therapy should be immediately administered.
d. Perform CT scan of the abdomen and pelvis. d. The patient has likely impending respiratory failure.
e. Prepare for urgent throacotomy. e. Vasopressors should be given to the patient.

10. A 22 year old male is assaulted in a bar. A semi-rigid cervical coller is applied, and 13. A 30 year old male is stabbed in the right chest. On arrival to ER, he is very short
he is immobilized on a spine board. On initial exam, VS are normal, GCS is 15. of breath. HR is 120 and BP is 80/50 mm Hg. His neck veins are flat. There is no
Which of the following is an indication for CT in this patient with possible minor diminished air entry on the right side, and there is dullness posteriorly on
traumatic brain injury? percussion. These findings are most consistent with:
a. Presence of hemotympanum. a. Tension pneumothorax.
b. Blood alcohol concentration of 0.16%. b. Pericardial tamponade.
c. Presence of an isolated 10 cm scalp laceration. c. Hypovolemia from liver injury.
d. Presence of a mandibuler fracture. d. Hemothorax.
e. History of assult. e. Spinal cord injury.

11. A 23 year old construction worker is brought to ER after falling more than 9 meters. 14. A specific aspect of the treatment of thermal injuries is:
VS: HR is 140, BP is 90/60, and RR is 36. He is complaining bitterly of lower a. Chemical burns require the immediate removal of clothing.
abdominal and lower limb pain, and his obvious deformity of both lower legs with b. Patients who sustain thermal injury are at lower risk of hypothermia.
bilateral open tibial fractures. Which of the following statements concerning the c. Patients with circumferential burns need prompt fasciotomies.
patient is true?
64
d. Electrical burns are associated with extensive skin necrosis (from entry 18. A 40 year old obese patient with GCS of 8 requires a CT. before transfer to CT
point to exit). you should:
e. The Parkland formula should be used to determine adequacy of a. Give more sedative drugs.
resuscitation. b. Insert a multilumen esophageal airway.
c. Insert a definitive airway.
15. A 15 year old male is brought to ER after being involved in a motor vehicle crash. d. Request a lateral cervical spine film.
He is unconscious and was intubated at the scene by EMS. On ER, O2 is 92%, e. Insert a nasogastric tube.
HR is 96 and BP is 150/85 mm Hg. Breath sounds are decreased on the left side
of the thorax. The next step is: 19. Lateral cervical spine films:
a. Immediate needle cricothyroidotomy. a. Must be performed in the primary survey.
b. Immediate needle thoracocentesis. b. Can exclude any significant spinal injury.
c. Chest tube insertion. c. Should be combined with clinical exam, AP and odontoid, CT.
d. Reassess the position of endotracheal tube. d. Are indicated in all trauma patients.
e. Obtain a chest X-ray. e. Require the following films: oblique views, AP, odontoid and flexion-
extension views prior to spinal clearance in trauma patients.
16. Which one of the following statements is true?
a. Elevated ICP will not affect cerebral perfusion. 20. A 30 year old male is brought to ER after falling 6 m. Flail chest on the right,
b. CSF cannot be displaced from the cranial vault. tachypneic and normal breath sounds. No hyperresonance or dullness. On oxygen
c. Cerebral blood flow (CBF) is increased when the PaCO2 is below 30 mm by face mask.ABG are: PaO2 of 45, PaCO2 of 28 and pH of 7.47. Abnormalities
Hg. in the patient's blood gases is due to:
d. AutoregulationCBF normally occurs between cerebral perfusion a. Hypoventilation.
pressures of 50 to 150. b. Hypovolemia.
e. Hypotonic fluids should be used to limit brain edema in patients with c. Small pneumothorax.
severe head injury. d. Pulmonary contusion.
e. Flail chest.
17. The first priority in the management of a long bone fracture is:
a. Reduction of the pain. 21. An 82 year old male falls down five stairs and presents to the ER. All are true
b. Prevention of infection in case of an open fracture. statements regarding his condition compared to a younger patient with similar
c. Prevention of further soft tissue injury. mechanism, except:
d. Control of hemorrhage. a. He is more likely to have had contracted circulatory volume prior to his
e. Improve long-term function. injury.

65
b. His risk of cervical spine injury is increased due to degeneration, 25. Hypertension following a head injury:
stenosis, and loss of disk compressibility. a. Should be treated to reduce ICP.
c. Intracranial hemorrhage will become sympotmatic more quickly. b. May indicate imminent herniation from critically high ICP.
d. His risk of occult fractures is increased. c. Indicates pre-existing hypertension.
e. His risk of bleeding may be increased. d. Mandates prompt administration of mannitol.
e. Should prompt burr hole drainage of potential subdural hematomas.
22. The most specific test to evaluate for injuries of solid abdominal organs is:
a. Abdominal X-ray. 26. Initial treatment of frostbite injuries involves:
b. Abdominal U/S. a. Application of dry heat.
c. DPL b. Rapid rewarming of the body part in circulating WARM water.
d. Frequent abdominal examination c. Debridement of hemorrhagic blisters.
e. CT of abdomen and pelvis. d. Early amputation to prevent septic complications.
e. Massage of the affected area.
23. A 14 year old female is brought to ER after falling from a horse. She is
immobolizedon a long spine board with a hard collar and blocks, cervical spine X- 27. Signs and symptoms of airway compromise include all of the following except:
rays: a. Change in voice.
a. Will show cervical spine injury in more than 20% of these patients. b. Stridor.
b. Will exclude cervical spine injury if no abnormalities are found on the X- c. Decreased pulse pressure.
rays. d. Dyspnea and agitation.
c. Are not needed if she is awake, alert, neurologically normal, and has no e. Tachypnea.
neck pain or midline tenderness.
d. Should be performed before addressing potential breathing or circulatory 28. Which one of the following statements is true regarding a pregnant patient who
problems. presents following blunt trauma?
e. May show atlanto-occipital dislocation if the power's ratio is < 1. a. Early gastric decompression is important.
b. A hemoglobin level of 10 g/dl indicates recent blood loss.
24. The most important consequences of inadequate organ perfusion is: c. The central venous pressure response to volume resuscitation is blunted
a. Multiple organ failure. in pregnant patients.
b. Decreased base deficit. d. A lap belt is the best form of restraint due to the size of the gravid uterus.
c. Acute gloumerulnephritis. e. A PaCO2 of 40 mm Hg provides reassurance about the adequacy of
d. Increased ATP production. respiratory function.
e. Vasodilatation.

66
29. A 30 year old is brought to ER after being injured in a motor vehicle crash. BP is 32. Supraglottic airway devices:
90/60 and HR is 122. She responds to the rapid infusion of 1 liter of crystalloid a. Are equivalent to endotracheal intubation.
solution with a rise in her BP to 118/ 88 and a decrease in her HR to 90. Her BP b. Require neck extension for proper placement.
then suddenly decreases to 96/66. The least likely cause of her hemodynamic c. Are preferable to endotracheal intubation in a patient who cannot lie flat.
change is: d. Are of value as part of a difficult or failed intubation plan.
a. Traumatic brain injury. e. Provide one form of definitive airway.
b. Ongoing blood loss.
c. Blunt cardiac injury. 33. A 70 year old male suffers blunt chest trauma after being struck by a car. On ER,
d. Inadequate resuscitation. GCS is 15, BP is 145/90, HR is 72, RR is 24 and O2 saturation on 5L is 91%.
e. Tension pneumothorax. Chest X-ray shows multiple rib fractures. ECG shows normal sinus rhythm with no
conduction abnormalities. Management should include:
30. Limb-threatening extremity injuries: a. Placement of a 22-french, right-sided chest tube.
a. Require a tourniquet. b. Serial troponins and cardiac monitoring.
b. Are characterized by the presence of ischemic or crushed tissue. c. Thoracic splinting, taping, and immobilization.
c. Should be definitively managed by application of a traction splint. d. Monitored IV analgesia.
d. Are rarely present without an open wound. e. Bronchoscopy to exclude tracheobronchial injury.
e. Indicates a different order of priorities for the patient's initial assessment
and resuscitation. 34. A 15 year old male presenting after a motorcycle crash. Initial exam reveals normal
VS. There is a large bruise over his epigastrium that extends to left flank. He has
31. A 29 year old female arrives in ER after being involved in a motor vehicle crash. no other appearent injuries. A CT of abdomen shows ruptured spleen surrounded
She is 30 weeks pregnant. She was restrained with a lap and shoulder belt, and by a large hematoma and fluis in the pelvis. The next step in this patient's
an airbag deployed. Which one of the following statements best describes the risk management is:
of injury? a. Splenic artery immobilization.
a. The deployment of the airbag increases the risk of fetal loss. b. Pneumococcal vaccine.
b. The use of seatbelts is associated with increased risk of maternal death. c. Transfer to pediatrician.
c. The mechanism of injury suggests the need for emergency C-section due d. Urgent laparotomy.
to the risk of impending abruption placenta. e. Surgical consultation.
d. The risk of premature fetal delivery and death is reduced by the use of
restraints. 35. A 30 year old male presents with a stab wound to the abdomen. BP is 85/60, HR
e. The deployment of the airbag increases the risk of maternal abdominal is 130, RR is 25 and GCS is 14. Neck veins are flat, and chest exam is clear with
injury. bilateral breath sounds. Optimal resuscitation should include:

67
a. Transfusion of fresh frozen plasma and platelets. d. Intraperitoneal bladder injuries are usually managed definitively with a
b. 500 mL of hypertonic saline and transfusion of pRBSs. urinary catheter.
c. Resuscitation with crystalloid and pRBCs until base excess is normal. e. Urinary catheters should be placed in all patients with pelvic fractures
d. Preparation for laparotmy while initiating fluid resuscitation. during the primary survey.
e. Fluid resuscitation and angioemobolization.
39. Which one of the following physical findings does not suggest spinal cord injury as
36. Initial resuscitation in adult trauma patients should: the cause of hypotension?
a. Be with 1-2 liters of crystalloid, monitoring the patient's response. a. Priapism.
b. Use crystalloid to normalize BP. b. Bradycardia.
c. Use permissive hypotension in patients with head injury. c. Distended neck veins.
d. Be with anon-blood colloid solution. d. Diaphragmatic breathing.
e. Be a minimum of 2 L of crystalloid in all trauma patients prior to e. Ability to flex forearms but inability to extend them.
administering blood.
40. Cardiac temponade:
37. A 25 year old male is brought to ER following a bar fight. He has an altered level
of consciousness, opens his eyes on command, moans without forming a. Is definitively managed by needle pericardiocentesis.
discernible words, and localizes to painful stimuli. Which one of the following b. Is most common with blunt thoracic trauma and anterior rib fractures.
concerning this patient is correct? c. Is easily diagnosed by discovery of Beck's triad in the ER.
a. Mandatory intubation to protect his airway is required. d. Is indicated by Kussmaul breathing.
b. His GCS suggests a severe head injury. e. Requires surgical intervention.
c. His level of consciousness can be solely attributed to elevated blood
alcohol. First group ATLS questions..
d. CT is an important part of neurological exam.
e. Hyperoxia should be avoided. 1) triage concept : save more lives with available resources
2) a patient with gun shot , BP 70/0 , Chest tube drained 120 ml , chest sounds
38. Which one of the following statements regarding genitourinary injuries is true? normal. next step? –Laboratomy
a. Urethral injuries are associated with pelvic fractures. 3) persistent pneumothorax after placing chest tube. Diagnosis? -
b. All patients with microscopic hematuria require evaluation of the Tracheobronchial injury
genitourinary tract. 4) which of the following is not part of the initial assessment? - determining
c. Patients with gross hematuria and shock will have a major renal injury as incomplete,or complete neurological deficit
the source of hemorrhage. 5) echomosys in prenium , blood in mayatus ,what will you do? - retrograde
urethrogram
68
6) Class II shock: - normal BP 1. Which of the following is addressed in the secondary survey? Answer: Forearm
7) patient with minimal trauma to chest and tenderness,ABCDE are good, how fracture
to manage? - pain medication
8) old patient on B-blocker and Coumadin (warfarin), decreased BP, normal 2. A young male fallen from height with obvious flail chest. ABG shows pH of 7.47. what
pulse, which of the following is true? - excessive fluids cause is the cause of this abnormality? Answer: pulmonary contusion
cardiopulmonary failure. 3. Cushing's triad which occurs in cases of increased intracranial pressure? Answer:
9) indication for intubation? - maxilofacial injury Bradycardia with irregular respirations and isolated increase in SYSTOLIC BLOOD
10) moderate head injury- GCS 11, what to do? - CT scan+repeat GCS PRESSURE.
assessment 4. Trauma in pregnant women, clear fluid leakage from vagina is an indication for
11) Neurogenic shock except: neurologic deficit hospitalization.
12) Neurogenic shock management: vasopressor 5. blood at the external meatus  do RUG
13) initial step in multiple injured pt: ask the patient's name 6. Scenario of pelvic fracture, which statement is correct? Answer: vertical shear force
14) pt. with blunt trauma to abdomen.. Decreased BP, no external bleeding: with posterior ligaments disruption.
FAST 7. 12 year old boy sustained blunt abdominal trauma while playing football. FAST scan
15) DPL: most sensitive is positive. He is hemodynamically stable. What to do next? Answer: CT abdomen
16) unsucceful endotracheal intubation: surgical cricothyroidectomy 8. Old patient, had multiple rib fractures, splinting of the right chest, what to do? Answer:
17) anatomy: choroid plexus produce CSF in the lateral and third ventricle.. give analgesic.
18) failed intubation: bogie 9. Memorize indications for burn transfer ( eg> 10% surface area affected  transfer)
19) unconscious baby then awake then deteriorate: epidural 10. In comparison with young adults, elderly patients exhibit which of the following
20) pt fell and can't move lower limb with sensory problem: spinal shock regarding brain injuries? A- increased cerebral blood flow. B- less stretching of
21) laparotomy indication: CT with retroperitoneal air bridging veins C- less subdural hematomas D- less brain contusions E- les mobility
22) circumfrential burn: escharotomy with angular acceleration and deceleration.
23) Pediatric: flexible mediastinum 11. Which of the following will be missed by DPL? Answer: subcapsular hematoma of the
24) sacral sparing: good prognosis spleen (because it is a retroperitoneal organ)
25) most common cause of acid base problem in pediatrics: ventilation 12. Which of the following tests will evaluate the retroperitonium? Answer: CT
26) 80 kg male with 50% burn and received 1 L NS came after 3 hour.. What is 13. Patient tried to commit suicide with a rope (hyperextension of the neck), when he
the fluid per hour in the next 5 hours? 1400 ml/hr presented to ER he had hoarsness of voice and crepitation in the neck, what to do?
27) X-ray of ruptured aorta except: air in mediastinum A- needle cricothyroidotomy B- surgical tracheostomy in OR C- direct laryngoscopy
28) indication to anti-Rh in pregnant women except: gunshot wound to the foot and intubation. Answer: ????
29) pregnant lady with PV bleeding, initial assessment Airway.. 14. Burn victim with signs of inhalation injury (carboneacous material, singed eyebrows)
30) difficulty in respiration, loss of sensation in the foot?- possible cervical injury
 Intubate.

69
15. Burn victim, has circumferential burn, core temperature is 34 C. what next? A-
escharatomy B- rewarm C- oxygen mask (I think the answer is C oxygen mask
because you should follow the ABCD priorities)
16. Which of the followings is NORMAL in pregnancy? A-increased residual lung volume
B- decreased plasma volume C- decreased total RBC mass D- widened symphysis
pubis. Answer is D
17. Trauma patient was hypotensive then you gave him 1 L of crystalloid and now he is
alert and talking. Which of the following statements is correct? He has good cerebral
blood flow
18. Which of the followings is a contraindication for nasal intubation? A- depressed skull
fracture B- Le Fort III fractures. Answer is B (Le Fort III fractures causes cribriform
plate fracture)
19. Which of the followings will benefit from oropharyngeal airway? A- posterior
displacement of tongue B- laryngeal edema. Answer is A
20. Patient came with severe head injury GCS is 6 and has poor anal sphincter tone and
diagrammatic breathing. His hands are flexed across the chest. What is the cause of
his injury? A-isolated head injury. B-lumbosacral verterbral injury. C- Cervical
vertebral injury. Answer is C
21. Question about widened mediastinum
22. Question about spinal board? Remove after completing assessment or it leads to
decubetus ulcers ..
23. CSF is between? Arachinoid and pia matter.
24. Question about transfer? Transfer after stabilizing the patient.
25. Scenario with scalp laceration .. the priority was to stop the bleeding with direct
pressure. The other choices are all after the circulation.
26. Patient with head injury and systemic hypotension, what is the most probable cause
of his hypotension? A- bleeding in the brain B- systemic hypovolemia. Answer is B
27. Dorsal column? Carries proprioception from the same side of the body.

70

S-ar putea să vă placă și